Спасёт ли вас прочный трос, или ваша участь предрешена?



Никто ещё не смог, и не сможет избежать последствий своих решений.
— Альфред А. Монтаперт


Отправлять свои вопросы и предложения в мою колонку могут все, но получить ответ на них посчастливится лишь избранным. На этой неделе честь достаётся юзеру klooloola, который хочет уточнить существование возможностей бегства из чёрной дыры. Фотон, конечно, не сможет из неё вырваться, но может быть, сможет что-то другое, если мы устроим всё следующим образом:

Мне интересно, возможно ли выползти из чёрной дыры. Не разгоняясь до скорости убегания, а используя некий гипотетический лифт. Тогда вам не придётся лететь быстрее света. Вам ведь не приходится преодолевать первую космическую скорость, поднимаясь на лифте. Большой корабль снаружи горизонта событий большой чёрной дыры с маленькими приливными силами мог бы подвесить человечка на тросе, опустить его за горизонт событий, а затем вытянуть обратно.

Интересная идея. Посмотрим, возможна ли она, или любой другой метод.



Чёрная дыра – это не просто сверхплотная и сверхмассивная сингулярность, в которой пространство так сильно искривлено, что всё, что упало туда, не может убежать. Обычно мы так её себе представляем, но чёрная дыра, если быть точным, это регион пространства вокруг таких объектов, из которого не может выйти ни материя, ни энергия – даже свет.

Ничего особенно экзотичного тут нет. Если взять Солнце как оно есть, и сжать его до нескольких километров в диаметре, вы получите именно чёрную дыру. Хотя Солнцу не грозит опасность такого превращения, существуют звёзды, которые именно так и превратятся в чёрные дыры.



Самые массивные звёзды Вселенной – звёзды с массой, превышающей солнечную, в двадцать, сорок, сто, или даже, в центре суперкластера на фото выше, в 260 раз – самые голубые, горячие и яркие объекты. Также они быстрее всех сжигают ядерное топливо в своём ядре: всего лишь за один-два миллиона лет вместо миллиардов, как это делает Солнце.

Когда у звезды кончается горючее, ядра атомов внутри неё подвергаются огромному гравитационному давлению: настолько сильному, что без ответного давления излучения, возникающего во время ядерного синтеза, они схлопываются. В менее экстремальных ситуациях в ядрах и электронах скапливается столько энергии, что в результате синтеза они превращаются в связанную массу нейтронов. Если масса ядра в несколько раз превышает массу Солнца, эти нейтроны получаются настолько плотными и массивными, что также коллапсируют и превращаются в чёрную дыру.



Имейте в виду, что это минимальная масса чёрной дыры: несколько солнечных масс. Чёрные дыры могут вырастать гораздо больше, путём слияния, поглощения материи и энергии и перемещения в центр галактики. В центре Млечного пути найден объект с массой в четыре миллиона солнечных, вокруг которого вращаются отдельные звёзды, и который при этом не испускает никакого света ни на каких длинах волн.



В других галактиках могут существовать и более массивные чёрные дыры, в тысячи раз превышающие по массе нашу, и теоретического верхнего предела для их массы не существует. Но мы пока не упоминали два интересных свойства чёрных дыр, которые должны подвести нас к ответу на сегодняшний вопрос. Первое – что случается с пространством по мере увеличения массы чёрной дыры.

По определению чёрной дыры, ни один объект не может вырваться из её гравитационного притяжения, независимо от его скорости, даже если она равна скорости света. Эта граница между тем местом, где объект может убежать, и тем, где не может, известна, как горизонт событий, и он есть у каждой дыры.





Это может вас удивить, но кривизна пространства гораздо меньше на горизонте событий вокруг самых массивных чёрных дыр, и гораздо больше вокруг менее массивных. Представьте следующее: если бы вы «стояли» на горизонте событий чёрной дыры, и ваши ступни находились бы как раз на границе, а голова – где-то на 1,6 метра дальше от сингулярности, на ваше тело действовала бы растягивающая его сила. Если бы это была чёрная дыра в центре нашей Галактики, эта растягивающая сила составляла бы всего лишь 0,1% от земной гравитации. А если бы Земля превратилась в чёрную дыру, и вы бы встали на её горизонте событий, то растягивающая сила была бы в 1020 раз сильнее земного притяжения!



В таких бы условиях нам пришлось проверять предположение читателя. Конечно, если эти растягивающие силы настолько малы на границе горизонта событий, они не должны быть сильно больше внутри него, и поэтому, учитывая электромагнитные силы, удерживающие твёрдые объекты, возможно, мы могли бы подвесить объект снаружи горизонта событий, пересечь его, а затем вытянуть объект назад.

Но возможно ли это? Чтобы разобраться, давайте вернёмся к происходящему на самой границе между нейтронной звездой и чёрной дырой: на границе необходимой массы.



Представьте, что у вас есть невероятно плотный нейтронный шар, с поверхности которого всё же может убежать фотон, вместо того, чтобы обязательно упасть обратно на звезду. Разместим ещё один нейтрон на её поверхности, и внезапно ядро потеряет возможность сдерживать гравитационный коллапс. Но отвлечёмся от мыслей о происходящем на её поверхности, и представим, что происходит внутри региона формирования чёрной дыры.

Представьте отдельный нейтрон, состоящий из кварков и глюонов, и представьте, что глюонам для переноса взаимодействий нужно перемещаться от одного кварка к другому.



Один из кварков окажется ближе к сингулярности в центре чёрной дыры, чем другой. Для обмена взаимодействиями – и для стабильности нейтрона – глюону в какой-то момент придётся переместиться от ближнего кварка к дальнему. Но даже на скорости света (а глюоны не имеют массы) это невозможно! Все нулевые геодезические, то есть пути, по которым проходит объект, движущийся со скоростью света, ведут к сингулярности в центре чёрной дыры. Более того, они никогда не отодвигаются от него дальше, чем в момент начала пути.

Поэтому нейтрону внутри горизонта событий чёрной дыры приходится коллапсировать и становиться частью сингулярности в центре.



Возвращаемся к примеру с тросом. Когда любая частица пересекает горизонт событий, для неё уже невозможно вернуться назад – даже для света. Но именно фотоны и глюоны нужны для переноса взаимодействий с частицами, находящимися снаружи горизонта событий – а они туда попасть не могут!

Это не значит, что трос порвётся. Скорее всего, сила притяжения затянет в дыру весь ваш корабль. Конечно, силы в определённых условиях не разорвут вас, но это не та причина, по которой стремление к сингулярности становится неизбежным. Это невероятная сила притяжения и факт, что частицы любой массы, энергии и скорости могут лишь направляться к сингулярности сразу после пересечения горизонта событий.




И, к сожалению, по этой причине невозможно выбраться из чёрной дыры после пересечения горизонта событий. Спасибо за прекрасный вопрос, и надеюсь, вам понравилось объяснение! Присылайте мне ваши вопросы и предложения для следующих статей.
Поделиться с друзьями
-->

Комментарии (224)


  1. DrZlodberg
    12.08.2016 14:49

    То есть сила притяжения нас не порвёт, но даже свободно падающий трос на границе будет нарезан на атомарные (или даже из элементарных частиц) ломтики?


    1. Murmand
      12.08.2016 14:58

      А вот у меня в голове не может уложиться то, что у горизонта событий у сверхмассивной ЧД меня не порвет в клочья, а ЧД которая получилась бы из Земли превратит мою тушку в атомарную пыль.


      1. DrZlodberg
        12.08.2016 15:09
        +3

        Тут как раз всё просто. Градиент разный. Чем ближе горизонт, тем резче переход.


        1. lolmaus
          14.08.2016 14:29

          Поясню, что чем черная дыра меньше по массе, тем меньше (ближе к центру черной дыры) будет горизонт.


          Поэтому маленькая дыра порвет вас как Тузик грелку Юпитер комету.


          А с огромной черной дырой приливные силы (т. е. дельта градиента притяжения) на горизонте будут намного слабее межмолекулярных связей, и вы попадете за горизонт, ничего не почувствовав.


          1. Capineiro
            14.08.2016 15:25
            +1

            Порвет то в любом случае, только в большой дыре уже далеко под горизонтом


            1. ankh1989
              15.08.2016 06:45

              С чего вы взяли? За горизонтом ведь может быть что угодно. Скажем, форма пространства-времени там может быть такой, что долететь до центра нельзя.


      1. 3cky
        12.08.2016 15:20
        +4

        Это результат сложения трех факторов: 1) радиус черной дыры (расстояние от центра до горизонта событий) прямо пропорционален ее массе, 2) сила гравитации на фиксированном расстоянии связана с массой объекта линейно, 3) сила гравитации убывает обратно пропорционально квадрату расстояния до центра объекта. В результате увеличение массы ЧД в два раза увеличивает ее размер тоже в два раза, но гравитация на горизонте событий при этом становится в два раза меньше.


        1. Smitak
          12.08.2016 17:36
          +1

          А мне тогда интересно: А что определяет физический диаметр горизонта событий и собственно чёрной дыры, если вся материя там сжимается практически в одну точку?


          1. Gallifreian
            12.08.2016 17:44
            +2

            «Размера» у черной дыры может и не быть, а вот диаметр Шварцшильда определяется массой, зарядом и угловым моментом.


          1. 3cky
            12.08.2016 19:23
            +1

            Ну классическое определение горизонта событий — это расстояние, на котором вторая космическая скорость становится равной скорости света.


          1. Mad__Max
            13.08.2016 20:19

            Про сжимание в точку — это всего лишь ничем не подтвержденная гипотеза. Следующая из математики, но скорее всего показывающая только одно — сфера применимости известных законов и соответствующих формул на горизонте ЧД уже заканчивается и дальше их использовать бесполезно. Нужно что-то получше (но пока ничего лучше не придумали).

            Ну а горизонт это вообще не физический объект, а чисто математическое/геометрическое понятие.


        1. MiB_K
          12.08.2016 20:16

          3) сила гравитации убывает обратно пропорционально квадрату расстояния до центра объекта. В результате увеличение массы ЧД в два раза увеличивает ее размер тоже в два раза, но гравитация на горизонте событий при этом становится в два раза меньше.


          Гравитация вроде должна стать меншьше не в 2, а в 4 раза. Разве нет?


          1. 3cky
            12.08.2016 20:28
            +2

            Это при неизменной массе, а у нас масса выросла в два раза, поэтому 4 нужно поделить на 2.
            Приливные силы, кстати, еще быстрее убывают, потому что они обратно пропорциональны кубу расстояния.


        1. Murmand
          12.08.2016 20:37
          +2

          Для меня все еще непонятно как (если представить пространство двумерной мембраной) на краю одной воронки из меня сделают «атомный фарш», а на краю другой я смогу вольготно прогуливаться, при этом в обоих случаях ступи я за край, возврата не будет.


          1. 3cky
            12.08.2016 21:04
            +2

            Ну представьте масляные пятна на поверхности ванны и бассейна. Если вытащить пробки там и там, так, чтобы в воде возникли воронки, то при одинаковой кривизне поверхности воды масляное пятно в бассейне будет находиться дальше от центра, чем в ванне. При этом скорость вращения в ванне для той же самой кривизны поверхности будет выше, так что пятно разорвет на части, в бассейне же оно будет некоторое время сохранять свою форму, пока не окажется слишком близко к сливу.


          1. Mad__Max
            13.08.2016 20:28

            Вольготно «прогуливаться» (или полетать хотя бы) в любом случае невозможно — там нет никакой точки опоры, по которой можно было бы «прогуляться» (на которую опираться / за которую держаться). В любом случае засосет внутрь и возврат назад невозможен.

            Разница между маленькими ЧД и гигантскими ЧД только в том, что малая ЧД гарантированно убьет (разрушит любой объект) еще задолго до достижения ее горизонта событий. Тогда как в случае с гигантской ЧД есть шансы добраться до самого горизонта и возможно даже пересечь его все еще оставаясь живым (для автомата — исправным) к этому моменту. При условии что вокруг ЧД очень «чисто» (нет какой-то другой материи которую она всасывает и которая при этом будет крутиться по орбите недалеко от горизонта на околосветовых скоростях).
            Гигантская ЧД все-равно потом все поглощенное покрошит в «атомный фарш» — просто это произойдет уже после пересечения горизонта событий, а не до этого.


            1. simki
              13.08.2016 21:12

              Гигантская ЧД все-равно потом все поглощенное покрошит в «атомный фарш» — просто это произойдет уже после пересечения горизонта событий, а не до этого.


              Внутри ЧД как-раз самое «безопасное» место во вселенной. Если удастся пересечь горизонт событий каким-то образом. Ближе к центру будет невесомость, в больших ЧД плотность «материи» намного ниже плотности воздуха, никаких ужасных давлений и излучений как в нейтронных звездах (вот там покрошит в субатомный фарш). Ну это если не углубляться в сингулярность конечно.


              1. Capineiro
                13.08.2016 21:27

                а как вы собираетесь в неё не углубляться?) (невесомость = свободное падение)


              1. Mad__Max
                13.08.2016 21:31

                Эта средняя плотность аналогична «средней температуре по больнице». Не известно сжимаются ли внутренности в сингулярность или же сжатие остановится на какой-то стадии и в центре например комок кварк-глюонной плазмы болтается, но никакого равномерного распределения этой плотности по объему ни в одной теории не предполагается.


                1. ankh1989
                  13.08.2016 22:53

                  Мне кажется, что кварк-глюонная плазма в центре это как то топорно. Почему бы не предположить, что никакого супер плотного вещества внутри нет, но есть хитро искривлённое пространство-время, что снаружи ч.д. имеет конечный и небольшой размер, а изнутри до центра лететь десятки мегапарсек?


                  1. Mad__Max
                    13.08.2016 23:32

                    Ну тогда надо придумать куда денется это сверхплотное вещество. Потому что рождение ЧД со сгустка сверхплотной материи и начинается и изначально в центре ЧД оказывается именно оно.

                    А что там дальше происходит и как это «изнутри» выглядит — можно что угодно напридумывать, хоть полностью другие физические законы которые начинают действовать внутри. Научных теорий вменяемых на этот счет нет. Экспериментальное исследование невозможно в принципе. В результате чего не придумать — не подтвердить, ни опровергнуть не получится.


                    1. simki
                      15.08.2016 04:25

                      Образование ЧД дыры начинается из центра. Из точки. Аналогия с «большим» взрывом. Большим взрывом для того пространства. И парадоксом для нашего.


                      1. Capineiro
                        15.08.2016 09:13

                        Нет, где-то рядом с радиусом Шварцшильда оно начинается. В самом центре коллапсирующей звезды пространство не искажено


                      1. black_semargl
                        15.08.2016 12:33
                        +1

                        Нет, просто средняя плотность вещества в неком объёме превышает критическую — и оппа, свету уже не улететь.
                        Просто для вещества массой со звезду даже нейтронная еле-еле, но вот газовое облако галактической массы уже может и само, без промежуточного звёздообразования.


      1. Gallifreian
        12.08.2016 15:26

        Слышали про комету Shoemaker-Levy 9? Она распалась именно из-за приливных сил Юпитера.

        Тут также объясняется откуда берется формула приливных сил.


      1. SkyBB
        12.08.2016 15:37
        +12

        Просто у сверхмассивной ЧД горизонт событий далеко от центра, поэтому дельта сил тяготения между ногами и головой маленькая. Грубая аналогия — если греть ноги от солнца или от костра, лёжа головой дальше от источника, то, добившись одинаковой температуры пяток (т.е. горизонта событий), в первом случае голова будет греться практически так же, а во втором — замерзнет.


  1. siryoshka
    12.08.2016 15:37

    Так информация не потеряна, энергия\масса успешно преодолела горизонт событий, как я понимаю в достаточно большей черной дыре плотность частиц равна примерно земной атмосфере.
    Я неспешно снимаю скафандр…


    1. Aversis
      12.08.2016 16:06
      +1

      Ссекундочку, я записыывааааюююууу…


    1. Capineiro
      12.08.2016 21:06
      +1

      Это если в среднем считать под горизонтом. Непосредственно рядом с её центром, я бы и в скафандре не пожелал бы оказаться.


    1. quwy
      15.08.2016 11:47

      Для внешнего мира — потеряна, назад пути нет. Там, в принципе, может быть целая звездная система с планетами и зелеными человечками, которые и не в курсе, в какой заднице Вселенной они оказались. Но тем, кто сидит снаружи, этого всего не существует, космическая цензура запрещает знать.


      1. simki
        15.08.2016 14:24

        Может и наоборот, у них хорошо и комфортно, никто посторонний не залетит в ближайшеи 100 трлн. лет — мой дом, моя крепость. Это нам надо беспокоится, то сверхновые рядом, то нестабильность Солнца, то еще что-то.


  1. AdvanTiSS
    12.08.2016 15:49

    Не проще ли объяснить тем, что когда сила гравитационного притяжения становится больше чем молекулярные силы притяжения, ваш трос станет рассыпаться на молекулы и частицы, сразу же, как только он начнет пересекать горизонт событий?


    1. Structure
      12.08.2016 16:35

      Если я правильно понимаю, то силы молекулярного притяжения ни на что не влияют в данном случае.


    1. koshi
      12.08.2016 16:35
      +1

      Нет, для падающего в дыру наблюдателя не будет заметен переход через границу. Трос не распадётся на молекулы, а элементарно порвётся где-то на отдалении от чёрной дыры.


      1. Structure
        12.08.2016 16:43

        Трос не порвется, если ЧД большая.


    1. khrundel
      15.08.2016 11:55
      +1

      Там же написано: в дыру упадёт корабль.
      Горизонт событий — это просто расстояние разграничения, при котором может или не может улететь нечто со скоростью света. Также можно провести вокруг чёрной дыры воображаемую сферу, которую может покинуть свет, но не могут покинуть объекты, движущиеся со скоростью меньше 0.99999*C. И сферу, которую не могут покинуть объекты со скоростью меньше 0.99*C.
      Корабль, имеющий массу, не может летать со скоростью света. Так что, после того как он приблизился к горизонту событий он уже попал в область из которой ему не вылететь, он может послать наружу свет или радиосигнал, но сам уже не выберется.


  1. NeoCode
    12.08.2016 16:01
    -1

    Интересно то, что если у сверхбольших черных дыр нас не порвет в клочья, но тем ни менее внутри макроскопического объекта при пересечении границы движение частиц и проводимость возможна только в половине всех направлений. Любые механизмы скорее всего встанут, любые электронные схемы перестанут работать (ток по проводникам не пойдет в половине направлений), а живая материя скорее всего очень сильно повредится и вряд ли сможет дальше функционировать как живая.
    Единственное что возможно поможет — как можно быстрее перейти горизонт событий (за микро или даже нано секунды). Чем меньше время — тем меньше деструктивное воздействие. Интересно, прав я или нет?


    1. Gallifreian
      12.08.2016 16:10

      Скорее всего время будет замедлятся пропорционально приливным силам — вас просто послойно унесет в будущее.


    1. custos
      12.08.2016 16:48

      Там вообще ничего не понятно. Если рассуждать как предложено в статье… Допустим один кварк нейтрона оказался за горизонтом событий, глюоны уже не в состоянии его связывать с остальными кварками, следовательно нейтрон должен развалиться. Однако нельзя вырвать кварк из нейтрона, глюонное поле породит ещё одну кварк-антикварк пару. С другой стороны нейтрон не может провалится целым за горизонт событий, поскольку радиус Шварцшильда для массы равной нейтронной звезде, примерно равен третьей части её радиуса, т.е. нейтронная звезда должна ещё не слабо сплющится, чтобы у неё появился горизонт событий.


    1. AdvanTiSS
      12.08.2016 16:50
      +2

      Мне кажется, что после пересечения горизонта, до встречи с дырой, тело будет находиться в состоянии свободного падения, тоесть невесомости относительно ЧД. Соответственно движение частиц относительно падающего тела вероятно не будет подвержено ограничениям, наложенным гравитационными силами ЧД.


    1. alexzzam
      12.08.2016 19:17

      Под горизонтом всё ещё хуже. То есть, это не отсекающая всё подряд плёнка, а просто внешняя граница области, в которой любое взаимодействие может случаться только от краёв к центру.


      1. boblenin
        12.08.2016 22:28

        От краёв к центру относительно попавшего внутрь ЧД или относительно центра ЧД?


        1. alexzzam
          13.08.2016 14:27

          К сингулярности. К центру, то есть, сферы Шварцшильда.


    1. Capineiro
      12.08.2016 20:57

      В свободном падении вы будете падать в дыру вместе с окружающим пространством. Так что локально вы не заметите переход. Можно даже пытаясь улететь от ЧД замедлить падение, но только не остановить. Конец наступит глубоко под горизонтом почти у самого центра из-за приливных сил.


  1. san-x
    12.08.2016 17:02

    А разве движение «от сингулярности» не продолжает быть невозможным после пересечения горизонта событий? Другими словами — «полупроводимость» сохранится и после его пересечения, так что просто «побыстрее пересечь горизонт событий» — видимо, не поможет.
    Другое дело, что ускорение и скорость движения «к сингулярности» не обязаны быть высокими (особенно у сверхбольшой ЧД и неподалеку от ГС).
    Это наводит на мысль, что если перед пересечением ГС разогнаться чуть выше «максимальной необходимой скорости взаимодействия» в направлении сингулярности — то как при пересечении ГС, так и внутри его, распространение взаимодействия снова будет возможно в любом направлении (относительно объекта). Другое дело, что любая электрика потребует разогнаться выше скорости света, но вот что-нибудь гидравлическое или чисто механическое — вроде как вполне сможет функционировать.
    А вот для живого организма — какая максимальная скорость взаимодействия?


    1. Murmand
      12.08.2016 21:38

      Почитал все коменты и мне кажется, что любое тело упавшее в ЧД на любой допустимой скорости просто превратится в поток элементарнейших частиц т.к. электроны будут сорваны с орбит вокруг атомов, да и сами атомы распадутся на составляющие составляющих.


  1. gerahmurov
    12.08.2016 17:34
    +1

    Вот тут задумался. А если к чёрной дыре подойдёт другая чёрная дыра, так что горизонты событий станут внахлёст, не будет ли бОльшая чёрная дыра вытягивать частицы из более маленькой? Или частицы — нет, просто маленькая дыра неразрывно всосётся в большую?

    Или наоборот, маленькая будет всасываться в большую медленно, но так как у маленькой градиент резче, она будет вытягивать из большой частицы в себя?


    1. yar3333
      12.08.2016 18:59

      Произойдёт искривление горизонта событий — он перестанет быть сферическим. Там где будет пересечение — силы гравитации начнут компенсировать друг друга и горизонты дыр подвинутся к центрам этих дыр.


      1. gerahmurov
        12.08.2016 19:02

        А что будет происходить с частицами на месте предыдущей границы горизонта? Они утянутся в след за горизонтом событий, или таки выберутся наружу, пусть и в другой горизонт?


        1. Capineiro
          12.08.2016 20:48

          Все частицы находятся в центре дыры в бесконечной малой точке с бесконечной плотностью. Это и называется сингулярностью. А горизонт событий — это как условная шуба. Сначала сольются горизонты, а потом под общим горизонтом полетят друг к другу сингулярности. Но как две бесконечно малые точки друг друга найдут уже совсем непонятно.


          1. gerahmurov
            12.08.2016 21:19

            А как же прилетающие частицы и квантовые флуктуации? Или на дыру не распространяются?


            1. Capineiro
              12.08.2016 21:27

              Для внешнего наблюдателя они визуально испытают красное смещение в ноль, застывая и чернея у горизонта. Со своей точки зрения — полетят как ни в чем ни бывало дальше к центру.


        1. stalinets
          12.08.2016 21:25

          Чёрные дыры сольются, горизонт событий снова будет шаром, а выступающие части исчезнут, превратившись в мощнейший всплеск гравитационных волн.


    1. alexzzam
      12.08.2016 19:09

      Сольются почти мгновенно.


    1. black_semargl
      13.08.2016 20:43

      Горизонты не могут быть внахлёст — радиус результирующей ЧД равен сумме радиусов сливающихся.


  1. saboteur_kiev
    12.08.2016 17:41
    +1

    Меня никак не покидает ощущение, что вся материя рано или поздно поразбегается по ближайшим черным дырам, затем они сбегутся в одну большую кучку, создадут мегачерную дыру, которая наберет некую критическую массу и случится очередная сингулярность.

    То есть где-то там на микроуровне, глубже кварков, есть что-то еще, что под большим давлением выдаст энергию, превышающую силу гравитации, либо то, на чем собственно гравитация работает перестанет корректно работать в какой-то квант времени, которого хватит на сингулярность.

    Кстати, тут же возникла мысль и вопрос для Итана — кто может его задать? Звучит примерно так:

    Если такое, как антигравитация в принципе существует, как бы оно повлияло на молекулярные связи и на объекты микроуровня (атомы, кварки, etc)?

    А то обычно антигравитацию рассматриваем только в макромире, в виде двигателей для кораблей.


    1. Smitak
      12.08.2016 17:56
      +1

      Вот тут предположения насчёт «супербольшой» чёрной дыры и нового Большого взрыва из неё порождают другой вопрос: А почему, собственно взорвалась та первоначальная сингулярность, а не «продолжала существовать» в виде чёрной дыры?
      А что, если никакого взрыва и небыло в той вселенной, там она остаётся чёрной дырой, но внутри неё существует наша вселенная и её внутренняя реальность для нас — Большой взрыв и возникновение вселенной? Тогда то что для нас — БЗ, для постороннего наблюдателя — коллапсирование звезды в чёрную дыру.


      1. upsilon
        12.08.2016 19:39

        Это рассуждение совсем не ново, не знаю, занимался ли этим кто-то именитый, но ко мне такие мысли еще в школе приходили.
        Данные рассуждения выходят за рамки существующих моделей, и при этом не помогают ничего объяснить за пределами черной дыры (а внутрь мы заглянуть не можем), так что пока что это просто фантазии.


  1. dmitry_dvm
    12.08.2016 18:10

    А эти джеты из черных дыр как тогда выстреливают, если ничто не может оттуда выйти? У них скорость выше скорости света?


    1. Structure
      12.08.2016 18:20
      +2

      Джет не из ЧД начинается.


    1. Alex_ME
      12.08.2016 18:25
      +2

      Джет состоит из вещества (?) и излучения, которые еще не попали за горизонт событий, но были разогнаны ЧД до больших скоростей.


  1. Arxitektor
    12.08.2016 18:29

    А можно ли выйти на орбиту вокруг сингулярности?
    под горизонтом событий?


    1. alexzzam
      12.08.2016 19:14

      Судя по конусам на последней картинке — нет.
      Ну, только если мы точечный объект на круговой орбите ровно по горизонту событий. Но точечных не бывает.


    1. dernasherbrezon
      12.08.2016 19:18

      Прочитал как «можно ли выйти на работу вокруг сингулярности». Похоже пора в отпуск.


    1. napa3um
      12.08.2016 20:16

      Если допустима любая сложность орбиты, то вы уже на ней. Вся материя рано или поздно падает в чёрную дыру.


      1. boblenin
        12.08.2016 22:38

        Почему вся? Часть материи уже улетела далеко и т.к. вселенная расширяется шансов ее вернуть — нет. Т.е. большая черная дыра в центре и улетевшие в бесконечность частички материи на периферии.


        1. SCareAngel
          15.08.2016 11:52

          Вселенная: Доктор, я центра не чувствую.
          Доктор: А у вас его нет!


    1. viatro
      15.08.2016 11:47

      Зарегистрировался специально, чтобы ответить.
      В 2011 наш ученый показал, что для вращающихся или заряженных черных дыр такое может быть, хотя орбиты и будут далеки от окружностей. Вот ссылка на препринт статьи: arXiv:1103.6140


  1. chabapok
    12.08.2016 19:38

    Как это:

    (1) «минимальная масса чёрной дыры: несколько солнечных масс»

    соотносится с этим:

    «если бы Земля превратилась в чёрную дыру»

    С учетом того, что масса земли меньше массы солнца.

    Насколько помню статью в википедии, существование черной дыры возможно и в малых массах, лишь бы соблюдалось уравнение. Так же, когда запускали коллайдер, то некоторые побаивались, что он будет производить сверхлегкие черные дыры. При этом никто возможность существования сверхлегких ЧД не опровергал — и этот факт свидетельствует в пользу того, что ваше утверждение (1) является неверным.

    Так же, в википедии упоминается теоретическая возможность существования ЧД массой 10^(?5) г.


    1. Ocelot
      12.08.2016 20:11
      +4

      Несколько солнечных масс — минимальная масса чд, которая может возникнуть самостоятельно при гравитационном коллапсе. Если сжимать вещество внешними силами, никто не мешает создать чд хоть массой в килограмм (правда, она очень быстро испарится).


      1. Idot
        12.08.2016 20:52

        «Быстро» это на сколько? Помню что Итан рассказывал, что время испарения обычной чёрной дыры намного превышает возраст вселенной, и даже когда потухнут все красные карлики, она всё ещё не успеет испариться. Так что «быстро» это сколько? Вдруг это «быстро» измеряется сотнями миллионов лет? Хотелось бы расчёт скорости испарения для такой массы.


        1. Capineiro
          12.08.2016 21:09

          Скорость испарения зависит от кривизны горизонта. Чем дыра меньше, тем она кривее и быстрее испаряется


        1. Mad__Max
          13.08.2016 21:26
          +1

          Это нужно Излучение Хокинга смотреть.

          У обычной ЧД — например массой 3 солнечных (самые маленькие из ЧД) излучение(испарение) будет как у тела с температурой 2*10^-8 — т.е. лишь миллиардные доли градуса от абсолютного нуля. В результате такая ЧД в текущих условиях вообще не испаряется — температура космоса вокруг намного выше из-за реликтового излучения: около +2,7 градусов по абсолютной шкале. Т.е. ЧД будет поглощать излучения больше чем излучать и поэтому наоборот будет постепенно расти вместо того чтобы испаряться.
          Но даже если бы этого фона не было и она висела в абсолютной пустоте при абсолютном нуле, при такой маленькой температуре испарение излучением действительно займет почти бесконечное время — намного больше времени чем возраст вселенной.

          А для ЧД массой в 1 кг температура излучения будет 10^23 градусов — на 30 порядков выше чем у «звездной» ЧД. Еще и интенсивность излучения пропорциональна 4й степени температуры. В результате это будет не просто быстрое испарение — а взрыв в котором вся масса практически мгновенно превратится в излучение и энергию разлетающийся частиц на скоростях близких к скорости света.
          Такой взрыв по силе будет намного больше чем от атомной или водородной бомбы аналогичной массы и приближаться к взрыву аналогичного количества антиматерии.
          Поэтому такие ЧД и не могут существовать — они в принципе крайне нестабильны.


          1. ankh1989
            13.08.2016 22:56

            Кстати почему такой взрыв будет мощнее? Ведь в атомной бомбе заряд превращается в излучение — всякие там гамма кванты. Разве можно этот заряд ещё эффективнее взорвать?


            1. Mad__Max
              13.08.2016 23:05

              Там только довольно небольшая часть массы в энергию превращается. В идеальной атомной бомбе — всего около 0.1% от массы заряда. В идеальной водородной несколько раз(2-3) больше.
              В реальных — наоборот в несколько раз меньше.
              Тогда как «взорвавшаяся» (очень быстро испарившаяся) ЧД должна предположительно десятки % своей массы в виде энергии выкинуть. Ну а антиматерия ~100%.


              1. black_semargl
                14.08.2016 07:07

                Антиматерия выдаёт 200% массы.
                Потому как аннигилирует с той же массой материи.


      1. chabapok
        12.08.2016 20:58

        Так в том то и дело, что в утверждение (1) ни о каком возникновении речь не идет. Там речь идет о существовании.
        Поправьте, это некорректно.


        1. Mad__Max
          13.08.2016 21:08

          Хех. А как возможно существование чего-либо, если это что-либо никогда не возникало? :)
          Если невозможно возникновение — невозможно и существование.

          А пока никаких других механизмов возникновения ЧД кроме как коллапса звезд нам неизвестно — значит существуют только ЧД звездной массы и выше.
          Чисто теоретически могут существовать и меньшей массы, например возникшие изначально при рождении нашей вселенной в момент Большого Взрыва. Или скажем при столкновении ультрарелятивистких струй (джетов). Но пока никаких свидетельств в пользу того, что такие реально существую нет. И даже никто не смог придумать механизма (просчитать физику и математику) как такие могли бы образоваться.


          1. black_semargl
            13.08.2016 22:26

            Даже при столкновении струй вряд ли возможно возникновения вещества плотностью выше атомного ядра.
            А любая мелкая неоднородность, теоретически способная стать зародышем — испарится быстрее чем что-то поглотит.
            Вышеупомянутая тобой черная дыра с массой один килограмм имеет радиус около 10-27 м — при диаметре атомного ядра 10-15.
            Т.е. хрен в неё что-то попадёт…


            1. Mad__Max
              13.08.2016 22:59

              Ну я и написал — чисто теоретически. Какого-то фундаментального запрета что такое не может происходить — нет. Но на практике нереально.
              Хотя в случае джетов надо еще учитывать что в хорошем джете все его содержимое летит очень близко к скорости света — так что на любые объекты, в т.ч. «зародыш» ЧД будет действовать сильное замедление времени резко снижающее скорость ее распада(испарения). Уже даже в земных ускорителях добивались и реально наблюдали замедление скорости распада (не ЧД конечно, но разных нестабильных частиц) в десятки и сотни раз. И теоретических ограничений насколько сильным может быть это замедление — нет. Все зависит от того до какой энергии (скорости) разогнать. На что способны джеты ЧД мы пока не знаем. Но из космоса иногда прилетают частицы таких энергий, что наши ускорители на этом фоне детские игрушки — значит где-то существуют какие-то очень эффективные и мощные «ускорители». Хотя не факт что это именно джеты.

              Так что если сложить сильное замедление времени (увеличивающее время жизни микро ЧД) и то что она летит почти со скоростью света через плотный поток частиц, т.е. хоть вероятность столкновения с одной конкретной частицей очень мала, но частота с которой она будет с ними встречаться — огромна. То можно предположить что какие-то мелкие ЧД все-таки могут сформироваться и успеть набрать какую-то массу.
              Но вот до макроскопических размеров вырасти им скорее всего уже невозможно — по мере роста массы она будет терять скорость, замедление времени будет уменьшатся и в какой-то момент она начнет испаряться быстрее чем поглощать новые частицы.

              «Реликтовые» ЧД более реальны — теоретически со времен БВ могли остаться ЧД промежуточной массы — намного меньше звезд, но достаточно тяжелых чтобы не испаряться слишком быстро. Сейчас их пытаются искать в данных телескопов как раз по излучению/испарению (пока без результатов).


          1. chabapok
            13.08.2016 23:49

            «Если невозможно возникновение — невозможно и существование.»
            Нет, возможно.

            Например. Вы на рыбалке бывали когда-нибудь? Возникновение возможно только маленьких рыб, а существование — и маленьких и больших. Точно так же, только наоборот (из больших — в маленькие), у черных дыр. Автор некорректно сформулировал мысль, не оправдывайте его.


            1. Mad__Max
              20.08.2016 15:06

              Автор в отличии от вас знает что в нашей вселенной маленькие ЧД из больших образоваться просто не могут. Сейчас ЧД могут только расти в размере. Так что чтобы существовали маленькие ЧД, они должны изначально сразу маленькими сформироваться. И поэтому невозможность образования = невозможности существования.


              1. simki
                20.08.2016 19:23

                Теоретически можно ЧД окружить сферой охлажденной до нуля, не пропуская в ЧД ни одного фотона. Только это долго и энергозатратно, наверно в 10^30 раз затратнее чем энергия ЧД.


                1. Mad__Max
                  20.08.2016 22:20

                  Ну теоретически можно. Но это повлияет только на возможность появления маленьких ЧД в очень далеком будущем. Но никак не на их существование в настоящем.
                  Даже если бы какая-то другая цивилизация зародившаяся и развившаяся намного раньше нашей начала этим заниматься еще миллиарды лет назад — все-равно значительно «похудеть» такая изолированная ЧД просто не успела бы — для этого даже всего текущего возраста нашей вселенной мало.

                  Так что чтобы маленькая ЧД могла существовать сейчас — она должна была изначально примерно такой же маленькой и сформироваться.Теоретически такая возможность допускается — их называют первичными черными дырами
                  Но на практике даже косвенных признаков того что они реально сущесвуют пока не обнаружено. В отличии от классических ЧД образовавшихся при коллапсе и слияниях массивных тех.


                  1. black_semargl
                    21.08.2016 21:16

                    Эти реликтовые ЧД по-идее должны быть рассеяны по вселенной равномерно — потому как протогалактические газовые облака им импульс передать никак не могли и затянуть в себя соответственно тоже.
                    Т.е. они практически все во внегалактическом пространстве — а там найти ЧД массой с Юпитер нашими средствами без шансов



    1. Jogger
      13.08.2016 02:15

      Ну следует читать «если бы земля, не изменяя размеров, увеличила свою массу до нескольких солнечных и стала бы чёрной дырой».


      1. censor2005
        15.08.2016 11:48

        По моему, следует читать «если бы Земля, под действием неведомых внешних сил, превратилась в черную дыру»


  1. Arxitektor
    12.08.2016 19:43
    +1

    Имеет ли наука представление о том что находиться под горизонтом событий?
    Горизонт представят собой 2-х мерную поверхность не имеющей толщины?
    При прохождении материя полностью распадается разрушаются даже элементарные частицы?
    превращаясь в какую то форму энергии?


    1. zookko
      12.08.2016 20:28
      +1

      Только фантазии и гипотезы, никаких знаний нет. И могут ли вообще быть?


    1. Capineiro
      12.08.2016 20:36

      Свободнопадающий в черную дыру наблюдатель не заметит никакой поверхности или особенных эффектов. Он вполне будет видеть другого наблюдателя, падающего на 10 метров впереди. Но внешний достаточно далекий наблюдатель их уже никогда не увидит. Еще по мере приближения к горизонту, 360 градусов звездного неба будет сплющиваться до 180 (и еще сильнее за горизонтом)


      1. boblenin
        12.08.2016 22:41

        А как можно увидеть падающего впереди если отраженный от него свет должен будет двигаться в направлении от сингулярности?


        1. izobr
          13.08.2016 01:26

          Фотонам не нужно лететь назад, летящий позади сам их догонит. Вы же можете слышать сидящего впереди вас пассажира в самолёте, летящем со сверхзвуковой скоростью.


          1. VenomBlood
            13.08.2016 02:14

            Вообще то в самолете воздух внутри тоже движется со скоростью самолета, так что его сторость иррелевантна.


        1. cyberly
          13.08.2016 02:33

          Скорость света вроде должна быть одинакова в любой системе отсчета (читал давным-давно в детской книжке, могу ошибаться). То есть свет от падающего впереди будет с точки зрения летящего за ним распространяться «как обычно».


      1. DoctorManhattan
        12.08.2016 23:01

        Как это соотносится с тем, что все геодезические ведут в центр черной дыры и нет никаких направлений, кроме как в центр с тем, что для того, чтобы увидеть падающего на 10 метров впереди, нужно поймать фотоны, которые уходят по направлению от центра черной дыры.


        1. Capineiro
          13.08.2016 07:14

          Для разных систем отсчета геодезические выглядят по разному. Если падающий посветит назад фонариком, то относительно него фотоны полетят назад, а относительно ЧД не полетят.


        1. Mad__Max
          13.08.2016 21:43

          Относительно центра ЧД эти фотоны НЕ будут двигаться назад. Они будут просто будут падать к центру намного медленнее чем падающие наблюдатели. Практически стоять на месте если вектор точно от центра направлен. Это 2й наблюдатель падая на них «наткнется» (налетит) на них.

          При этом с точки зрения наблюдателя (в его системе отсчета) это свет будет лететь к нему как обычно — с его точки зрения какой-то разницы по сравнению с тем как такое происходит вдалеке от ЧД не будет.

          Ну это все исходя из предположения, что теория относительности вообще применима к тому что происходит за горизонтом ЧД. Что совсем не факт. Скорее наоборот.


          1. DrZlodberg
            15.08.2016 08:28

            Кстати интересно. Везде обсуждается, что ничто не может перейти горизонт в обратную сторону, но нигде не упоминается, что за горизонтом гравитация ведь должна быть ещё больше. Т.е. происходящее на границе можно (можно ведь?) экстраполировать дальше до сингулярности. А вот как это повлияет на все рассуждения выше и ниже — это непонятно.


            1. simki
              15.08.2016 14:31

              Огромная черная дыра может состоять из разряженного газа. Просто газовое облако, которое в силу размеров стало черной дырой. Внутри наблюдатель не заметит никаких изменений. Гравитация там будет ниже, чем на Земле от Юпитера, например.


              1. DrZlodberg
                15.08.2016 15:16

                В теории. Если газ будет достаточно разогрет (иначе он схлопнется в сингулярность).
                Но речь то не о сверхмассивных, а о любых. Да и разогретый газ вряд-ли будет находиться сразу за горизонтом. Для этого он должен быть чертовски сильно разогрет. Вероятнее всего материя лежит далеко внизу относительно горизонта, и вот что там между горизонтом и «далеко внизу» как раз интересно.


            1. Mad__Max
              16.08.2016 01:30

              В существующих проверенных теориях гравитации (ОТО) гравитация еще сильнее быть не может — как раз на горизонте ЧД она достигает теоретического максимума.
              А как строить новые теории, которые имели бы более широкую область применимости, в т.ч. за горизонтом ЧД — непонятно. Мы же даже теоретически не можем какую-то информацию получить о том, что там происходит — не на чем базировать теории. Можно много чего придумать (нафантазирововать), но без экспериментальной проверки (хотя бы косвенной) это все не научные теории, а так и остается фантазиями.

              Если начать тупо экстраполировать не обращая внимания на вылезающие нестыковки, то по-идее получится что объекты внутри могут превысить скорость света (что противоречит базе теории, которую мы пытаемся для экстраполяции использовать), а замедление времени сменится обратным его ходом.


              1. simki
                16.08.2016 04:46
                +1

                Или мнимые величины появятся, минус под корнем. Непонятно что это значит.


              1. DrZlodberg
                16.08.2016 08:57

                Хм. Про теоретический максимум не разу не попадалось. Всегда пишут про горизонт и сингулярность, которая где-то под ним. А вот что творится между ними…
                А где можно почитать что-нибудь адекватное, на эту тему?


                1. Mad__Max
                  17.08.2016 23:24

                  Никто не знает что под горизонтом. Т.е может быть еще выше гравитация и может быть, но совершенно не понятно что тогда происходит с самим пространством при этом. По ОТО уже на горизонте его искривление достигает максимума, а течение времени останавливается. Дальше считать невозможно — сплошные деления на 0 и корни из отрицательных величин. Да и представить сложно — что будет с пространством если его искривлять уже после достижения максимума искривления? Может «порвется»? Или просто свернется замкнувшись само не себя и потеряв связь с остальным пространством?
                  Что будет со временем если оно уже стоит, но продолжать еще сильнее замедлять? Куда же дальше — уже и так нулевая скорость… Пойдет в обратную сторону? Останется таким же «замороженным»? Ну и прочие подобные веселые вопросы, для ответов на которые нет даже зацепок или намеков.
                  Никаких адекватных теорий на этот счет мне не попадалось…


  1. Capineiro
    12.08.2016 20:16

    При свободном падении через горизонт достаточно большой дыры ваше кровообращение не остановится от того, что кровь будет течь только в сторону дыры. Лететь от дыры можно. Нельзя улететь. Аналогия — вы на весельной лодке пытаетесь избежать ниагарского водопада. Можно грести относительно воды и плавающих рядом щепок, можно замедлить падение, но невозможно избежать бездны.


    1. upsilon
      12.08.2016 21:19

      1)> Лететь от дыры можно
      Что?
      2) Одна часть тела прошла через горизонт, другая нет. От первой части ко второй ни материя, ни излучение не может пройти ко второй по определнию черной дыры. Поэтому для Вас это должно быть равносильно тому, что Вас разорвали.
      3)Смысл аналогии?


      1. Capineiro
        12.08.2016 21:42

        Для внешнего неподвижного наблюдателя космонавт не разрывается, а краснеет (от красного смещения), а потом чернеет и вплющивается в горизонт. Если космический корабль падает в дыру, то космонавт который летит недалеко впереди может вернуться обратно в корабль. Поскольку он незаметно для себя сплющился из-за лоренцевого сокращения, для его системы отчета горизонт еще где-то впереди. А для внешнего наблюдателя они уже пересекают горизонт.


        1. upsilon
          13.08.2016 22:48

          Вижу веры больше, чем фактов в Вашем сообщении.


  1. lavmax
    12.08.2016 21:56
    +3

    И почему эти статьи оставляют больше вопросов, чем дают ответов.


    1. boblenin
      12.08.2016 22:43

      И насколько, наверняка, проще было общаться с физиками 19 века :)


  1. stargazr
    12.08.2016 23:01

    Разве космонавт на тросе не будет спускаться к горизонту событий бесконечно долго из-за гравитационного замедления времени?


    1. HappyLynx
      12.08.2016 23:31

      С точки зрения стороннего наблюдателя — да. С точки зрения космонавта — нет.


      1. ankh1989
        13.08.2016 04:01

        Кстати, как это будет выглядеть со стороны? Ведь чтобы наблюдать этого космонавта бесконечно долго, от него должны лететь фотоны всё это время и мне не понятно откуда они будут браться если сам космонавт давным давно улетел.


        1. Capineiro
          13.08.2016 05:56

          Тут просто всё, фотоны от него будут всё больше краснеть. Поэтому количество излучаемой космонавтом энергии для внешнего наблюдателя будет стремиться к нулю.


        1. Mad__Max
          13.08.2016 21:58

          Со стороны это должно выглядеть так: Чем ближе к горизонту тем выше замедление времени у него (и всех внутренних процессов) и тем выше красное смещение любого излучения исходящего от него.
          В результате фотоны будут приходить все реже и энергия каждого фотона будет становиться все меньше. Скачала уйдет в инфракрасное излучение, потом в радиоволны, а потом их вообще станет невозможно принять. С точки зрения внешнего наблюдателя он станет неотличим от самой поверхности черной дыры.

          Браться они ни откуда не будут — это те же самые фотоны что излучал падающий космонафт в последние моменты перед пересечением горизонта событий. Скажем излученное за последнюю секунду (с точки зрения падающего космонафта) перед пересечением горизонта потом строны будут ловить еще несколько тысяч лет пока энергия фотонов не станет настолько малой, что их невозможно зарегистрировать никакими методами. Хотя все эти тысячи лет космонафт (точнее то что от него осталось) находился уже за горизонтом.

          P.S.
          Бесконечного времени скорее всего вообще не будет. Бесконечности возникают из упрощенных математических моделей где горизонт ЧД считается абсолютнонеподвижным. Тогда падение на него займет бесконечное время (с точки зрения внешнего наблюдателя). Но любая падающая в ЧД масса, увеличивают массу этой самой ЧД. А с ростом массы растет ее радиус пусть и немного. В результате сам горизонт чуть-чуть сместится навстречу падающему объекту и поглотит его за большое, но конечное время.


          1. upsilon
            13.08.2016 22:59

            К пострикптому бы добавил, что существующие математические модели могут просто не работать вблизи (а тем более внутри!) черной дыры, поэтому не стоит слишком серьезно относиться к таким мысленным эксперементам.
            Вроде бы это почти очевидно, но не уверен, что все читающие это понимают.


            1. simki
              14.08.2016 02:40

              Там основные процессы идут согласно теории относительности, ничего такого, что противоречило бы современным теориям.


  1. Tsimur_S
    12.08.2016 23:01

    Но если из горизонта событий ничто не может выйти, то как его покидает гравитация ведь она же является волной распространяющейся со скоростью света?


    1. stargazr
      12.08.2016 23:23

      Может, потому, что на нее не действует гравитация?


    1. ankh1989
      13.08.2016 04:03

      Возможно гравитация возникает не внутри ч.д. а на сфере её окружающей.


    1. Capineiro
      13.08.2016 07:34

      сама гравитация не является волной так же как и электромагнитное поле (волны — это возмущения этого поля а не само поле)


  1. Arxitektor
    12.08.2016 23:17

    Как я понимаю возмущение от искривления идет со скоростью света.
    то есть если тело исчезнет то другие заметят исчезновение его гравитационного поля расстояние/ скорость света


    1. Capineiro
      13.08.2016 07:30

      Информация не может быть передана наружу горизонта, так что видимо гравитационные волны не будут выходить наружу. А сама гравитация никуда не денется. Если сделать ЧД из одних протонов, она останется положительно заряженной, но посигналить изнутри, меняя распределение протонов тоже не удастся.


      1. kauri_39
        13.08.2016 11:29

        Но всё-таки гравволны выходят от сингулярностей в центрах ЧД. Зарегистрировали же эти волны от слияния двух ЧД, при котором масса 3 Солнц превратилась в энергию гравволн.
        Что-то не ладно в нынешней гравитационной теории. Диаметр ЧД, горизонта событий или сферы Шварцшильда определяется массой её сингулярности. Всё, что движется со скоростью света и медленней, не может выйти из-за этой сферы и падает на сингулярность. Но если гравволны от «дефекта масс» в 3С при слиянии 2-х сингулярностей в одну как-то выгребли из-за сферы Шварцшильда — горизонта новой ЧД, то их скорость там была выше скорости света? И с ней они должны путешествовать дальше?


        1. Capineiro
          13.08.2016 14:59
          +1

          image
          Дыры сделали полсотни оборотов за одну секунду перед слиянием. А как горизонты соприкоснулись, волны утихли за сотые доли секунды.


          1. kauri_39
            13.08.2016 20:01

            Спасибо, это графики, показывающие амплитуду и частоту гравволн. Но меня интересует теоретический вопрос: почему эти волны, имеющие световую скорость распространения, преодолевают пространство от сингулярности в центре ЧД до горизонта событий и выходят из-за него дальше?

            И возник ещё вопрос: почему при прохождении гравволны через плечи интерферометра возникает рассогласование фаз отражённых в них лучей света? До волны лучи приходили на детектор в противофазе и гасились, фототока не было. Но расширение пространства в одном плече и его сжатие в другом соответственно увеличивает и уменьшает длину световых волн, не меняя их фазы! Волна удлиняется и при космологическом расширении пространства…


            1. Capineiro
              13.08.2016 21:05

              От сингулярности никаких волн не выходит. Каждая ЧД порождает однородное гравитационное поле. Волны появляются вокруг, когда эти черные дыры начинают мельтешить в хороводе. Так-то система Солнце-Юпитер тоже гравиволны порождает, хоть и ничтожные.


            1. Mad__Max
              13.08.2016 22:27

              Эти волны НЕ от сингулярностей внутри ЧД. Эти волны излучала система из 2х ЧД непосредственно перед слиянием (большая часть приведенного графика) и в процессе слияния (последние 10-15 миллисекунд, увеличенный кусочек в конце графика).

              Как только 2 ЧД стали одним целым — волны сразу же прекратились. Мы не можем сказать (не знаем) было ли дальше после этого сближение 2х сингулярностей уже внутри одной общей ЧД и их слияние. И существуют ли сингулярности вообще. Но если существуют и сливались — то волны от них уже не смогли выйти наружу, как и предсказывали модели — детекторы после объединения горизонтов событий уже ничего не зафиксировали.
              Скорость грав. волн подтверждена на уровне ~ световой и косвенно и прямо.

              Интерферометр работает не за счет растяжения-сжатия пространства проходящей волной. Там реально (физически) двигаются зеркала хоть и на крайне малые расстояния. Ну точнее т.к. это циклические колебания вокруг нулевой точки — то не двигаются, а слегка вибрируют — с частотой гравитационной волны, но очень малой амплитудой. Хотя это больше подход как рассматривать — классически или через ОТО. Можно и через сжатие-растяжение пространства: изменения длины волны достаточно — расхождения длин волн 2х лучей света которые изначально были равны при их интерференции дает картину аналогичную расхождению фаз при совпадающей длине волны. Они перестают идеально точно гасить друг друга и фотодектор регистрирует появление сигнала.


              1. kauri_39
                14.08.2016 01:51

                Да, понятно, что регистрируемые нами волны генерирует система из двух ЧД. Но сами ЧД — порождение сингулярностей, материи более плотной, чем материя нейтронных звёзд (материя кварковых звёзд?). Поэтому и гравволны танцующих дыр — это следствие танца сингулярностей перед их слиянием.
                Выручает гидродинамическая модель ЧД (из «Мембраны»), которая согласуется с моим механизмом гравитации. Пространство или эфир поглощаются сингулярностями со сверхсветовой скоростью за их горизонтами. Вращаются сингулярности, вращаются и их сферические горизонты. В любом направлении в плоскости вращения скорость движения эфира к паре сингулярностей переменная. В направлении поперёк пары скорость меньше, в направлении вдоль пары скорость выше. Такое «дёрганье» поглощаемого эфира и порождает в нём гравволны — от самого края сингулярностей в сторону горизонта событий и дальше за него. Но все волны между сингулярностью и горизонтом уходят вместе с эфиром в сингулярность, а волны от горизонта и дальше — дальше и уходят. Их мы и регистрируем.

                Отражающие зеркала в интерферометре двигаются вместе с пространством. Точнее, то расширяющийся, то сжимающийся эфир приводит в движение эти зеркала. Они попеременно удаляются от делительного зеркала и приближаются к нему. И соответственно длина волны растёт и падает. Будет ли при этом меняться фаза?
                Подключаем картинки Итана с его космологическим покраснением фотонов при расширении Вселенной. Шарик растягивается, и нарисованная на нём синусоида тоже растягивается. Её первый и последний гребень отдаляются друг от друга. Если бы с ними были связаны зеркала, то никакой смены фаз не было бы.
                Спасает, по-моему, то, что делительное зеркало неподвижное, связано с Землёй, и на него гравволна не действует. А на отражательные, подвижные зеркала, действует. Поэтому при растяжении пространства световые волны не только удлиняются, их ещё уносит от делительного зеркала, и на нём фаза волны, да, меняется.


            1. Smitak
              14.08.2016 17:32

              Насколько мне известно, скорость распространения гравитации примерно в 20 раз выше скорости света.


              1. kauri_39
                14.08.2016 20:21

                Откуда известно? Пока неизвестно, что такое гравитация, а тем более — какая у неё скорость распространения. У Ньютона она действовала мгновенно, у Эйнштейна — со скоростью света, но природу гравитации не знал ни тот ни другой, поэтому оба могут ошибаться насчёт её скорости.


                1. Smitak
                  14.08.2016 23:30

                  У Эйншейна она действовала со скоростью света, потому что он условно так определил эту некую аксиому, что ничто не может распространяться быстрее скорости света. Если погуглить, то можно найти достаточно косвенных доказательств, что гравитация не может быть равна скорости света и что она распространяется либо мгновенно либо в десятки-сотни-тысячи раз быстрее. Точные измерения пока что не проведены, ввиду их трудновыполнимости.


                  1. kauri_39
                    14.08.2016 23:53

                    Да, мне тоже кажется, что скорость распространения гравитационного взаимодействия практически мгновенна. При этом квадрупольные гравитационные волны, имеющие световую скорость, тоже существуют. Ведь в них есть поперечные перепады плотности эфира — как в поперечной электромагнитной волне. Поэтому и скорости обоих волн совпадают. Но в эфире есть и продольные волны — со сверхсветовой, почти мгновенной скоростью.


                    1. black_semargl
                      15.08.2016 02:05

                      Хохма в том, что гравитация не распространяется вообще.
                      Это электрические заряды рождаются и аннигилируют, а массы сколько возникло в момент БВ — столько и есть.
                      Она только меняет координаты.


                      1. kauri_39
                        15.08.2016 14:47
                        -1

                        Вы о какой массе речь ведёте? Если которая масса покоя частиц, то она тоже аннигилирует, превращается в энергетический эквивалент — фотоны, не имеющие массы покоя. И в слиянии 2 ЧД, впервые зарегистрированном, 3 солнечных массы ушло в энергию гравволн. Значит, гравитационный заряд может меняться — мгновенно у частиц и быстро у сингулярностей, и это порождает соответствующие гравволны.
                        Второй случай требует выяснения скорости распространения гравитации. Допустим, в центре галактики вращаются чёрные дыры, а вокруг них — звёзды. Дыры в итоге сливаются в одну — с огромным «дефектом масс». Гравволны ушли из центра, и скоро уйдут из галактики. Что и когда произойдёт со звёздами, которых удерживал массивный ранее центр? Их может выбросить из галактики, может отбросить на соседние звёзды. К подобным катаклизмам надо готовиться и знать скорость распространения гравитации.

                        Вероятно, гравитация — перемена силы гравитационного притяжения — распространяется мгновенно. Это если понимать гравитацию как поток эфира к поглощающей его материи.
                        С этим предположением трудно уживается существование квадрупольных гравволн со световой скоростью. Ну, возникли перепады плотности эфира (плотности энергии вакуума), почему они должны существовать миллиарды лет в упругой, средней по плотности среде? Почему они не сгладятся тут же до средних значений?
                        Все скажут: такова ОТО, она многократно проверена, и последний случай её подтверждения — регистрация гравволн. Вот Игорь Иванов из «Элементов» объясняет, почему гравволны регистрируются: http://igorivanov.blogspot.ru/2016/02/GW-acting-on-light.html
                        А у меня сомнения. В гравволне, в фазе растяжения, плотность эфира (плотность энергии вакуума) снижена, как и в гравполе. И если во втором случае длина световых волн растёт (из-за чего замедляется ход атомных часов), то должна она расти и в первом случае — у лазера. И тогда рост длины плеча интерферометра компенсируется ростом длин световых волн в нём. И никакого смещения фаз не будет. Или я в чём-то ошибаюсь?


                        1. Mad__Max
                          16.08.2016 02:15

                          Аннигиляция материи никакого значения не имеет. В ОТО энергия влияет на пространство-время точно так же как и масса. Поэтому если в какой-то точке часть массы преобразовалась в энергию, то для окружающей материи ничего не изменится — притягиваться к этой точке они будут с такой же силой как и раньше.

                          Так что как и в электричестве в гравитации «заряды» всегда сохраняются (возможно это правило нарушается только для вселенной в целом на космологических масштабах). Это же классический закон сохранения энергии, просто в другой формулировке — с точки зрения гравитации.

                          Понятие скорость «действия» к гравитации в общем-то не применимо. Просто нет такого понятия в ОТО — есть какая-то кривизна пространства-времени в данной конкретной точке и любая материя/энергия находящийся в этой точке или пролетающая через нее уровень этой кривизны всегда «чувствует» — независимо от того насколько далеко находится объект вызывающий это искривление.

                          Можно считать мгновенным действием если так проще понять — хотя это неправильно, т.к. испытывая эффекты гравитации идет взаимодействие не с исходным объектом (где-то далеко), а с самим искаженным пространством в этой конкретной точке. Поэтому расстояние всегда ~0 и взаимодействие ~мгновенное.

                          А вот любые изменения уровня гравитационного притяжения (кривизны П-В в ОТО) распространяются строго со скоростью света от объекта вызывающего эти изменения. В частности у вас в примере звезды «почувствуют» что что-то случилось с центральной ЧД галактики только через промежуток времени равный расстоянию / скорость света — одновременно с тем как до них дойдут гравитационные волны. А до этого будут взаимодействовать с искривленным пространством так, как будто с ЧД ничего не происходит. Потому что они взаимодействуют не с ней самой, а лишь со своим локальным куском пространства.

                          P.S.
                          Если свой эфир пытаетесь вписать не противореча уже существующим проверенным теориями и экспериментальным фактам, то скорость движения его частиц придется тоже ограничить скоростью света. Иначе совершенно непонятно почему электромагнитные волны или гравитационные волны передающиеся как колебания или потоки его частиц ограничены скоростью света. Если бы частицы могли двигаться быстрее — такого ограничения на распространения волн в такой не среде не было бы.

                          При этом на скорость расширения вселенной (как целого, а не отдельного взятого ее кусочка) это ограничений не накладывает. Если частицы эфира постоянно возникают по всему объему, то если взять достаточно большой объем — расстояния в нем могут увеличиваться быстрее скорости света, несмотря на то что никакие его частицы не движутся быстрее этой скорости.


                          1. kauri_39
                            16.08.2016 16:12

                            Да, я пытаюсь вписать свой эфир в реальность, потому что вижу, что ОТО не очень в неё вписывается. Реальность мне подсказывает, что в вакууме, имеющим свойства среды, должны быть продольные сверхсветовые волны — подобные звуковым «сверхповерхностным» волнам в воде. Однако ОТОшные физики на этот счёт молчат, как партизаны.

                            Молчат они и на вопрос, каким образом материя искривляет пространство вблизи и вдали от себя. Они этого не знают, природа гравитации им не известна. Поэтому я, например, сомневаюсь, что они верно объясняют высокие скорости вращения звёзд и галактик присутствием в их системах тёмной материи. А может, это тёмная энергия — расширяющийся эфир, вакуум, пространство — прижимает их к центрам их вращения?

                            В последнее время сомневаюсь, что физики смогли зафиксировать в гравитационном интерферометре гравитационные волны. Потому что измерять растяжения и сжатия пространства можно лишь такими «линейками», которые сами на них не реагируют, а таких в природе не существует. И как фотоны атомных часов реагируют на гравполе увеличением длины своей волны, так реагируют на фазу растяжения пространства и фотоны в лучах интерферометра.

                            Более того, как объясняют физики выход гравволн, имеющих световую скорость, от вращающейся пары сингулярностей, которые так искривляют пространство вокруг себя, что вокруг них образуется горизонт событий? Ведь не поверхность ЧД (горизонт событий) создаёт гравитацию ЧД, а сингулярность (кварковая звезда?) в её центре.

                            Вы верно полагаете, что объём эфира радиусом 14 млрд световых лет из-за повсеместно возникающих в нём новых эфиронов расширяется от условного наблюдателя в его центре (нас) со сверхсветовой скоростью. Таковы упругие свойства этой среды, проявляющиеся в образовании космологического горизонта. Но эти же свойства позволяют эфиру устремляться за гравитационные горизонты ЧД тоже со сверхсветовой скоростью. И я уверен, что эти свойства используют иные цивилизации в создании сверхсветовых кораблей.

                            Эфир — это упругая среда взаимно сжатых эфиронов (микровселенных, согласно эволюционной модели мира). По отдельности эти частицы не движутся со сверхсветовой скоростью. Соразмерные им частицы — фотоны (разумные микровселенные) — движутся в нём со скоростью света, создавая вокруг себя волновые «электро-магнитные» колебания эфира.
                            Такая картина не противоречит существованию почти мгновенных продольных волн в упругом эфире. Если, допустим, при поглощении фотона электрон увеличил свою массу (энергию), то всасывание им эфира резко увеличилось, и от него почти мгновенно распространится продольная волна разрежения эфира. Для этого взаимно сжатым эфиронам вовсе не обязательно с такой же скоростью придвинуться к электрону с возросшей массой. Им достаточно это сделать почти одновременно, как и полагается при распространении продольных волн.


                            1. Capineiro
                              16.08.2016 20:37

                              Безумно любопытно стало узнать больше о разумных микровселенных


                              1. kauri_39
                                16.08.2016 21:44

                                Легко: http://futurologi.livejournal.com/33983.html
                                Разумные вселенные любого масштаба — это единые для своих вселенных системы цивилизаций. В модели я называю эту форму организации материи федерациями. О существовании пятого измерения — бесчисленных масштабов пространства-времени в бесчисленных вселенных — можно прочесть в моей первой здесь публикации.


                                1. Capineiro
                                  16.08.2016 22:00

                                  Обязательно ли наличие в микровселенной разумной федерации? Возможны пустые квадраты в вашей эволюционной матрице по ссылке?


                                1. Capineiro
                                  16.08.2016 22:14

                                  Еще вопрос, проводили ли вы опыты по генерации/дектектированию продольных электромагнитных волн? Электродинамика это вроде как запрещает, но эксперимент бы подтвердил вашу правоту.


                                  1. kauri_39
                                    16.08.2016 22:45

                                    Надеюсь, что далеко не в каждой вселенной, и в микровселенной в частности, была организована полномасштабная эволюция материи с образованием федерации. Возможно, их большая часть изначально безжизненна, но есть и те, что не пройдут естественного отбора на статус разумной вселенной, способной контролировать плотность своей внутренней среды, когда начнётся взаимное сжатие вселенных.

                                    Я такие опыты не проводил. Поправка: электромагнитные волны — поперечные. Продольными мгновенными волнами должны быть гравитационные волны, возникающие при мгновенной перемене массы, например, электрона в момент поглощения/излучения им фотона.
                                    Возможно, что такие мгновенные волны излучаются возбуждёнными белковыми молекулами, на которых записаны образы в памяти человека. Тогда можно предположить их участие в коллективной мыслительной деятельности людей, в их телепатическом общении.
                                    Свои каналы мгновенного обмена информацией нам показывают запутанные частицы материи, начиная с фотонов — разумных микровселенных. Запутанность лишь выявляет наличие таких каналов, но они, согласно модели, сохраняются и при нарушении запутанности.


                                    1. Capineiro
                                      17.08.2016 08:54

                                      А какие трудные моменты есть в вашей модели? Есть ли вещи еще не нашедшие стройного объяснения? Или которые могут поставить под сомнение корректность вашей модели? Какие эксперименты в нашей вселенной могли бы подвердить вашу модель, а какие опровергнуть?


                                      1. kauri_39
                                        17.08.2016 15:51

                                        Есть трудные моменты. В космологии — как взаимное сжатие вселенных не приводит к смешению их упругих сред. Для решения этого вопроса нужно изучать свою Вселенную и взаимно сжатые вселенные предыдущего масштаба — её эфир, проявляющий квантовые свойства.
                                        Моя модель объясняет будущее человечества. Поэтому для её проверки можно дождаться исполнения важного этапа будущего: «второго пришествия Христа» — явления инопланетной акушерской миссии от нашей родительской цивилизации. Поскольку человечество уже представляет собой единый планетарный организм, то ждать осталось недолго.
                                        Но это футурологическая проверка модели. Есть и физическая проверка, которая частично уже выполнена. Это испытания «невозможного двигателя» — EmDrive. Они показали, что тяга у него есть, и что её невозможно объяснить традиционной физикой. Объяснение через эффект Унру в неё не вписывается, поскольку требует наличия у фотонов массы покоя.
                                        Моё объяснение — фотоны в резонаторе концентрируются у его широкого торца, и они ликвидируют эфироны. И самой ближней материей, которая начинает притягиваться к этой области пониженной плотности эфира, является широкий торец резонатора. Поэтому вся конструкция начинает двигаться в сторону узкого торца резонатора.
                                        Проверить это объяснение несложно. Достаточно прикрепить атомные часы снаружи широкого торца, они будут отставать в этом искусственном поле гравитации от часов в отдалении. Не будут отставать — моя модель наполовину окажется неверной.
                                        А как подкопаться к общему закону эволюции — не представляю. В известной части эволюции всё совпадает, а в нашей федерации — надо дождаться интеграции в неё.


                                        1. Smitak
                                          17.08.2016 16:57

                                          В вашей теории всё в кучу. Если вы объясняете свойства света, гравитации, устройство вселенной, то не нужно приплетать тут человечество как единый организм и т.д. Потому что получается что всё это мироустройство должно крутиться вокруг нас, а это уже средневековым религиозным мракобесием попахивает. Причём тут люди?


                                          1. kauri_39
                                            17.08.2016 18:44

                                            Вы, наверное, не знакомились с эволюционной моделью мира, ссылку на которую я дал выше. Поэтому Вам кажется, что «в моей теории всё в кучу», а это система идей и знаний.
                                            Мироустройство вовсе не крутится вокруг нас. Таких как мы человечеств даже в нашей Галактике не менее десятка. Цивилизации — это просто форма организации материи, седьмая по счёту, начиная от кварков. Для вселенской системы цивилизаций мы ещё не родились, мы пребываем на стадии изолированного, «внутриутробного» развития и пока не имеем собственного сверхсветового «зрения». Отсюда парадокс Ферми и прочее.
                                            Ознакомьтесь с моделью, и многое станет понятным.


                                        1. Capineiro
                                          17.08.2016 20:34

                                          В общем-то эксперимент доступный. Параметры EmDrive вполне известные. Заказать ведро у толкового токаря думаю не будет стоить баслословных денег, магнетрон добывается из микроволновки. Крутильные весы в этом эксперименте вообще не нужны. Остается добыть атомные часы. Погуглите symmetricom csac sa.45s. Думаю, эта часть будет дороговатой. Если брать две штуки (чтобы сравнивать показания), то это где-то 3000$ будет стоить (текущую цену лучше уточнять у производителей). В общем 4000$ по прикидке на все про всё… не что бы дешево, но даже если не получится эту гипотезу проверить, всё равно в итоге останется на руках EmDrive поиграться =)


                      1. Mad__Max
                        16.08.2016 01:46

                        С электрическими зарядами на самом деле тоже самое, что и с массой. После окончания БВ суммарный электрический заряд не меняется. Любые рождения частиц или их аннигиляция происходит парами и с противоположными знаками. Распады одиночных частиц происходят с сохранением заряда (например нейтрон с зарядом 0 распадается на протон с зарядом +1 и электрон с зарядом -1, мюон с зарядом -1 на электрон с зарядом -1 и нейтрино с зарядом 0 и т.д.). В результате суммарный заряд никогда не меняется.

                        И у электрического поля как и у гравитации так же нет никакой скорости — оно просто есть в определенной точке пространства с определенной напряженностью. Распространяются только градиенты этого поля (например из-за движения электрических зарядов в пространстве) как волна изменений этого поля — со скоростью света. Собственно свет и есть частный случай такой волны.

                        Единственная разница с гравитацией, что в электрических зарядов у нас есть два противоположных знака (+) и (-), а для гравитации похоже существует только один (+) — масса-энергия.


                        1. black_semargl
                          16.08.2016 04:15

                          Суммарный-то не меняется, но когда из ничего возникает пара разного знака — то их воздействие на окружающий мир начинает распространятся. Со скоростью света, да.
                          А гравитационная масса не возникает, она лишь меняет координаты.


                          1. Mad__Max
                            17.08.2016 23:43

                            Если они не начали разлетаться — то не распространяется. Т.к. с точки зрения окружающего мира электрический заряд в этой точке пространства как был 0 (нет частиц), так и остался 0 (есть 2 частицы с суммарным нулевым зарядом). И только когда они начнут двигаться относительно друг друга, например разлетаться в разные стороны — появятся изменения в электромагнитном поле.

                            Так же и с массой — только ее движение порождает волны гравитации. Аналогией рождения пар тут будет деление массы, т.к. пар или противоположных зарядов не существует. Была она точечная масса М, раскололась на 2 с массой М/2 каждая. Но пока они остаются вместе рядом — никаких волн или изменений гравитационного поля (искривления пространства вокруг них) нет — они идентичны одиночному заряду с массой М. И только если они начнут двигаться друг относительно друга — начнет меняться гравитационное поле и появятся волны в нем.

                            Т.е. и там и там на что-то влияет только движение зарядов (электрических или гравитационных). Разница разве что родившая пара античастиц не может НЕ начать двигаться после рождения. Они просто не могут существовать рядом неподвижно в отличии от гравитационных зарядов.


              1. sumrakssk
                15.08.2016 11:53

                Хм… источником не поделитесь? А то что-то я в этой жизни упустил.


  1. xilix
    12.08.2016 23:27

    Как-то концовка зажевана. Засосет корабль. Ну давайте избавимся от понятия корабль и натянем бесконечно прочный тросс между двумя черными дырами одинаковой массы. А с другой стороны прикрутим черные дыры саморезами к небесной тверди, чтобы они не сближались друг к другу. Тогда можно будет выбраться?


    1. Capineiro
      13.08.2016 06:02
      +2

      Это вопрос из разряда «что будет, если всеразрушающий молот ударит по абсолютно неразрушимой наковальне?». Наука такое не изучает.


      1. LifeKILLED
        13.08.2016 23:11

        Ну а такой вариант: две ЧД собираются схлопнуться. Корабль находится между ними в центральной точке. Мы отправляем зонд на тросе под горизонт событий. Если трос натягивается слишком сильно, смещаемся к противоположной ЧД, и этим действием не даём наш корабль утянуть в ЧД с зондом. Поскольку ЧД постоянно сближаются, горизонт событий со сторон друг друга тоже уменьшается (гравитация компенсируется). Как только уменьшение радиуса горизонта событий пройдёт через зонд и освободит его, можно будет принять с него на корабль сигнал с данными. Затем наш корабль пересылает эти данные уже к нам на базу (мы, ясное дело, этим кораблём управляли либо удалённо, либо на автоматике, а сами находились в стороне).


        1. Capineiro
          13.08.2016 23:33

          image
          Изоповерхность гравитационного потенциала, соответствующего горизонту, будет выглядеть как метабол. Т.е. корабль тоже окажется под горизонтом и мы в любом случае со стороны ничего не увидим


          1. LifeKILLED
            13.08.2016 23:36

            Это при очень близком сближении, фактически слияние. Если ЧД находятся дальше, то их горизонты событий наоборот уменьшаются со внутренней стороны


            1. Capineiro
              14.08.2016 00:25

              Нашел крайне любопытное моделирование, как раз в тему. Выглядит очень правдоподобно. http://samlib.ru/p/putenihin_p_w/tonnel.shtml
              Интересно, могут ли теоретически разлететься черные дыры после того как их горизонты соприкоснулись


              1. Mad__Max
                14.08.2016 01:23

                Не смогут. Моделирование абсолютно неправильное — там даже не учтено притяжение ЧД друг к другу (взаимное влияние гравитации друг на друга или взаимное искривление пространства если с точки зрения ОТО). Не говоря уже об игнорировании гравитационных волн.

                В реальности настолько близко подошедшие друг к другу ЧД не пролетят мимо друг друга «чиркнув» горизонтами, а выйдут на круговую орбиту вокруг общего центра масс и сделав всего несколько сходящихся витков (из-за излучения грав. волн) сольются в одну. «Космолет» так из под горизонта никогда не выскочит — сначала будет под горизонтом одной из ЧД, потом окажется под общим искаженным горизонтом сливающихся ЧД, потом останется под горизонтом новой единой ЧД выросшей почти в 2 раза в размерах. Не говоря о том, что еще перед этим он будет уничтожен теми самыми грав. волнами.

                А вообще это очень весело использовать ньютоновскую физику для моделирования черных дыр, которые с точки зрения ньютоновской физики вообще не существуют! :)


                1. Capineiro
                  14.08.2016 01:29

                  Моделирование там действительно крайне странное, но концепт очень красивый. Почему бы внутри правда не быть горизонту? Даже когда дыры слились уже, между сингулярностями в точке Лагранжа может оставаться крохотный кусочек нормального (относительно внешнего наблюдателя) пространства.


                  1. Mad__Max
                    14.08.2016 01:48

                    Хм, ну внутри действительно может быть кусочек пространства с относительно нормальными условиями и котором могут существовать сложные материальные объекты (вплоть до живых). На какое-то время.
                    Только из этого кусочка уже никогда не выбраться наружу и не рассказать/передать что там находится — а в этом весь смысл этой идеи и моделирования был — поставить под сомнение утверждение что из ЧД ничто не может вернуться.

                    Без него оно бессмысленно. Просто «нырнуть под горизонт и при этом не умереть в процессе» — можно получить гораздо проще. Находим одиночную сверхмассивную ЧД. Выбираем такую чтобы вокруг нее было максимально «чисто» (отсутствие какой-либо материи). Или «чистим» сами если не нашли подходящую. Ну и дальше просто летим в нее. Это несравнимо проще и вероятнее чем 2 сверхмассивные ЧД летящие откуда-то издалека и сталкивающиеся на идеальном расстоянии друг от друга и корабль выдерживающий идеальную дистанцию до центров обоих.

                    С точки зрения корабля при пересечении горизонта ничего особенного происходить не будет.
                    Если верно предположения о сингулярностях (т.е. вся масса ЧД сосредоточена только в ее центре, весь остальной объем пустой) то и после пересечения ничего не будет — до тех пор пока не подойдет слишком близко к самой сингулярности.


                    1. Capineiro
                      14.08.2016 02:18

                      Только из этого кусочка уже никогда не выбраться наружу и не рассказать/передать что там находится

                      Можно взорвать сверхновую в точке Лагранжа как только общий горизонт сомкнулся. Тогда черные дыры изменят импульсы и разлетятся, а сверхновая выберется)


                1. Capineiro
                  14.08.2016 01:39

                  А что мешает горизонту разорваться обратно на два (хоть и ненадолго), если дыры чиркнут?


                  1. Mad__Max
                    14.08.2016 01:54

                    Сами горизонтам — ничего не мешает. Просто сингулярности (которые и определяют горизонт) не могут так двигаться как показано — по прямым линиям с постоянной скоростью. Они будут притягиваться к друг другу, постоянно разгоняться из-за этого, а их траектория загибаться пока не перейдет в замкнутую кривую вокруг общего центра масс. Это без учета грав. волн — если учесть просто гравитацию хотя бы по Ньютону. В модели учтена действие гравитации обеих ЧД на корабль, но не учтено действие гравитации ЧД друг на друга.

                    Если добавить ОТО и грав. волны — вместо замкнутой кривой сингулярности начнут описывать сходящуюся спираль, а их горизонты уже никогда не оторвутся друг от друга.


                    1. Capineiro
                      14.08.2016 02:04

                      Прямые линии мне тоже глаз режут, но не в этом суть. Если у дыр такой расход на гравиволны, то можно же их предварительно разогнать до скоростей чуть меньше скорости света. Тогда они совершат гравитационный маневр и разлетятся чиркнув горизонтами (главное чтобы центр масс каждой из них не попал под горизонт дыры партнера). Вопрос в том, допускает ли физика разрыв горизонта после такого чирка


                      1. simki
                        14.08.2016 02:47

                        В принципе гравитационные телескопы могут такое засечь, кратковременный гравитационный импульс, без затухающих колебаний.


                      1. Mad__Max
                        15.08.2016 00:28

                        Сложно сказать — нужно кого-нибудь попросить полноценно со всеми эффектами ОТО попробовать просчитать подобную ситуацию.

                        Чтобы они после такого тесного сближения смогли разойтись двигаться навстречу друг другу они должны с очень высокими скоростями — на вскидку где-то не ниже 0.7-0.9 скорости света (такие скорости у сливающихся ЧД на последнем этапе когда они уже касаются горизонтами). При этом уже начинают сильно сказываться все эффекты ОТО, которые могут принципиально поменять картину процесса, а не просто уточнения небольшие дать.
                        Например при такой скорости кинетическая энергия сближающихся ЧД сравнима с их массой. И т.к. масса и энергия эквивалентны, эта энергия будет влиять на радиус/диаметр ЧД. При этом по мере сближения скорости будут возрастать (из-за взаимного притяжения), будет расти кинетическая энергия, поэтому будет расти и размер ЧД. А при попытке расхождении — все в обратную сторону (т.к. взаимное притяжение будет наоборот тормозить).

                        Т.е. даже размеры ЧД будут динамически меняться (а не только форма их горизонтов). И может получиться так, что при максимальном сближении (ему же соответствует макс. скорость и максимальная энергия) будут не только горизонты событий пересекаться, но и сами сингулярности окажутся под горизонтами друг друга из-за роста размеров ЧД и из-за этого «разлепиться» ЧД уже не смогут независимо от скорости сближения.
                        А может и нет — довольно сложные расчеты нужны чтобы проверить.


                        1. Capineiro
                          15.08.2016 09:06

                          Спасибо за интересный комментарий. На таких скоростях черные дыры видимо уже начинают накручиваться друг на друга из-за скручивания пространства между ними, это должно еще больше усложнить разлет.


      1. LifeKILLED
        13.08.2016 23:24

        «Наука такое не изучает»

        Наука изучает в том числе и такое. Сферические кони в вакууме, кот Шредингера, всяческие поезда со скоростью света. Большинство мысленных экспериментов, которые описывают границы расчётов, звучат именно так. Просто потому что это возможно с точки зрения математики.


        1. simki
          14.08.2016 02:49

          Когда-то мысленным экспериментом была и таблица умножения, радолампа, ракета, лазер, полупроводник, гипотеза Пуанкаре… ))


  1. simki
    13.08.2016 00:20

    Интересно, если джета черной дыры в центре нашей галактике, случайно повернется на некоторое время в сторону Земли, какие будут последствия? Долетит хоть что-то, или наоборот, сожжет всю планету?


    1. Capineiro
      13.08.2016 05:59

      Джет в центре направлен по нормали наружу. Как ось волчка. Больше бояться надо сверхновых. Мало не покажется.


      1. simki
        14.08.2016 02:43

        Да, например Эта Киля


  1. potan
    13.08.2016 12:27

    А если оказаться мехду двумя ЧД, можно ли выбраться из-под горизонта одной за счкт тяготерия второй?
    И что быдет с источником гравитационных волн за горизонтом — гравитационные волны могут передать информацию из ЧД?


    1. isden
      13.08.2016 14:03

      Так вторая космическая внутри горизонта больше скорости света будет же.
      А вот насчет грав.волн — вопрос интересный. По идее — это же колебания искривления пространства-времени, и ЧД их «засасывать» вроде как не должна, как я понимаю.


      1. isden
        13.08.2016 14:12

        Другое дело, что я как-то не представляю, как их генерить внутри сферы Шварцшильда. Изменять массу ЧД изнутри самой ЧД — нельзя. Быстро двигать относительно друг друга массивные тела внутри неё — тоже нельзя (двигать можно только в одном направлении — в сторону сингулярности). Вот если бы уметь как-то искривлять пространство, имея в наличии лишь одну энергию — то может быть (и то, при условии обеспечения работоспособности этого самого гипотетического убер-дивайса по искривлению).


        1. Mad__Max
          13.08.2016 23:26
          +1

          Тут выше обсуждали интересный мысленный эксперимент, которые и не совсем мысленным оказался. При слиянии 2х ЧД какое-то время после слияния у нее внутри под горизонтом событий будут летать 2 сингулярности или 2 каких-то других сверхплотных объекта типа кварковых звезд (ядра 2х исходных ЧД). Они должны продолжать создавать гравитационные волны — до тех пор пока сами не сольются в одно целое, как перед этим слились их горизонты событий.

          Смогут ли эти волны выйти наружу из ЧД? Теория говорит что вроде бы нет. При условии что наши теории вообще применимы к пространству за горизонтом (в чем есть обоснованные сомнения).

          Эксперимент — замеры слияния 2х ЧД на детекторах ЛИГО, когда впервые напрямую доказали существование грав. волн с теорией пока согласуются: после слияния горизонтов ЧД волны сразу же затухли (амплитуда упала до нуля или по крайней мере ниже чувствительности текущих детекторов).
          Так что либо грав. волны НЕ могут вырваться из ЧД так же как и свет. Ну либо в центрах ЧД никаких сингулярностей не существует и это лишь артефакт (глюк) нашего математического аппарата, а масса например каким-то образом «размазана» по всему объему ЧД (или по поверхности ее сферы, а внутри вообще ничего нет).


          1. isden
            13.08.2016 23:33

            > после слияния горизонтов ЧД волны сразу же затухли

            А вот как определили, что произошло слияние именно горизонтов (а не сингулярностей)? Как я понимаю, каких-то внешних эффектов быть не должно же. А мы можем только грав. волны детектировать от этого события.


            1. Mad__Max
              14.08.2016 02:29

              Ну просто смоделировали по ОТО и сравнили с экспериментальными данными — совпало очень хорошо. Сама модель довольно сложная, я сам не до конца ее понимаю(как впрочем и ОТО не до конца, меня нормально только на СТО хватает).
              Но чтобы конкретно этот момент понять достаточно будет частоты волн — она на прямую зависит от расстояния между центрами масс (допустим в ЧД у нас сингулярности и тогда этот центр это не просто математическая точка как центр Земли, а и физически вся масса там и собрана) и скорости движения. Чем ближе сближаются центры масс, тем выше становится частота волн которые они излучают.

              Ну и эксперимент показывает что после достижения частоты соответствующей расстоянию между центрами ЧД когда их горизонты начинают сливаться амплитуда волн начинает резко падать и быстро уходит ниже чувствительности детекторов — см картинку выше кто-то выкладывал.

              Грубо на пальцах можно даже прикинуть — сигнал обрывается после превышения частоты 400 Гц — это как раз примерно соответствует частоте вращения 2х масс порядка 10 солнечных каждая на расстоянии друг от друга порядка их гравитационных радиусов (которые и задают горизонты событий) со скоростью порядка 0.5 скорости света(скорость мы знаем из предыдущего моделирования их постепенного сближения). Точно правда не получится — т.к. надо учитывать то что горизонты в этот момент уже совсем не 2 сферы из себя представляют, а сильно искажаются + нужно учитывать другие эффекты ОТО.

              Но принцип должен быть ясен — сами сингулярности продолжая сближаться продолжали бы создавать грав. волны все большей и большей частоты. Причем если сингулярность классическая математическая (т.е. бесконечная малая, с бесконечной плотностью) — частота так же должна стремиться к бесконечности. Даже если не математическая, а просто очень плотные компактные объекты(кварковые звезды к примеру) в разы/на порядки меньше своих горизонтов — то все-равно частота волн должна была расти до гораздо больших значений.

              Кстати в этом случае по частоте и форме волн можно было бы сделать реконструкцию и примерно определить размеры и плотность этих сингулярностей внутри ЧД. Но природа такого подарка не сделала.

              Так что такие экспериментальные данные говорят об одном из 2х:
              1(вероятное) — из под горизонта ЧД не могут вырваться в том числе и грав. волны. Сингулярности продолжая сближаться их еще долго излучали с растущими частотами, но мы их не можем принять, т.к. они так и остались навсегда под горизонтом объединенной ЧД. Значит никакая информация ЧД не покидает.
              2 — внутри ЧД нет никаких сингулярностей, их масса распределена более-менее равномерно по объему и поэтому слияние горизонтов означает уже окончательное слияние ЧД — дальше уже нечему излучать грав. волны, т.к. имеем единый объект. А одиночные объекты в принципе не могут грав. волн излучать, нужна как минимум пара.


              1. black_semargl
                14.08.2016 07:34

                Волны выбраться не могут, но сама-то гравитация выходит. Поэтому объект находящийся рядом притягивается
                Но притягивается-то он не к центру масс, а к каждому из тел. Притом не к той точке где они в текущий момент, а с поправкой на дистанцию делённую на скорость света.
                А расстояния до ЧД разные — поэтому и точка к которой притягиваемся будет отставать на разное расстояние. И поэтому суммарный вектор будет не к центру тяжести системы, а отклоняться. И даже после того, как всё уйдёт под радиус.
                Аналогично и с падением электрически заряженного обьекта — притяжение к нему никуда не денется после падения. И куда именно притягивается наверно можно замерить.


                1. Mad__Max
                  15.08.2016 01:22

                  По текущим моделям и представлениям и гравитационное притяжение и электрическое (в случае наличия заряда) ЧД оказывает только как одно целое(как если бы вся масса/заряд были собраны точно в центре или наоборот равномерно распределены по всему объему) — объект находящийся на одном и том же расстоянии от горизонта будет испытывать одинаковые силы независимо от того, что происходит в этот момент под горизонтом или от того с какой стороны он к нему приближается.

                  Напрямую это проверить естественно невозможно. Но косвенно можно — в таком раскладе как у вас написано, при меняющейся силе притяжении (и векторе) из-за продолжающегося вращения сингулярностей под горизонтом на самом горизонте будут искажения(искривления его формы) — дыра будет не симметричной сферической, а иметь 2 «выпуклости» на поверхности(напротив каждой из сингулярностей), которые будут вращаться (выпуклость будет пробегать по поверхности горизонта) с той же частотой. И согласно ОТО эти «выпуклости» будут тоже излучать грав. волны — которые уже вполне можно принять и зарегистрировать.
                  Но их не зарегистрировали — в обоих слияниях ЧД которые пронаблюдали до этого момента сразу после слияния волны исчезают — значит ЧД принимает правильную симметричную форму, а все «выпуклости» разглаживаются сразу же после слияния.
                  Это процесс «разглаживания» и успокаивания поверхности только что образовавшейся после слияния ЧД называется Ringdown и происходит очень быстро — единицы — десятки миллисекунд для ЧД звездных масс.


                  1. black_semargl
                    15.08.2016 02:07

                    Заряд не может проскочить от поверхности до центра быстрее света.
                    и особенности этого проскакивания возможно могут дать информацию о внутренней структуре.


                    1. Capineiro
                      15.08.2016 09:19
                      +1

                      Заряженное тело скорее всего для внешнего наблюдателя размажется по поверхности горизонта, так как будет падать сквозь него бесконечно долго


                    1. Mad__Max
                      16.08.2016 02:36

                      А может быть и может. Нам скорость света «запрещает» превышать только ОТО. Но ОТО за горизонтом уже нормально не работает (не применима). Так что возможно за горизонтом возможно и сверхсветовое движение (или допустим сама скорость света меняется).

                      Ну даже если и сохраняется тот же лимит на скорость, то это чисто теоретическая возможность — даже не превышая скорость 1с но близко к ней (а что-то падающее в ЧД всегда развивает близкие к свету скорости) расстояние от поверхности до центра у ЧД звездых масс (типа тех что мы наблюдали — 7-20 солнечных) заряд проскочит за время всего порядка 0,0001с (0,1 мс). Дистанционно такие быстрые процессы не измерить, как собственно и колебания электрического поля от падающего заряда. Может быть когда-нибудь в очень далеком будущем, при условии освоения межзведных перелетов, да найдя дыру побольше типа центральных галактических и наблюдая ее с относительно небольшого расстояния в момент падения в нее заряженных тел…


    1. Capineiro
      13.08.2016 14:56

      По идее можно было бы свободно гулять по перешейку горизонтов в момент слияния, но космонавта там порвет на атомы теми же гравиволнами. Если смотреть на зарегистрированные графики, то в момент слияния гравитационные волны внезапно затухают за сотые доли секунды до нулевой амплитуды


      1. potan
        13.08.2016 23:12

        Но перешеек будет двигаться и горизонт событий может сместиться и освободить что-то, что под ним было.


        1. Capineiro
          13.08.2016 23:35

          Горизонт сместится со всем содержимым


      1. LifeKILLED
        13.08.2016 23:18

        Волны затухают по амплитуде, но частота их повышается, и резкость между переходами только растёт.

        Может быть, можно будет собрать устройство, которое окажется гораздо прочнее в том числе и на атомном уровне и сможет вовремя переслать полученные данные.


  1. Andrei_ra
    15.08.2016 11:47

    Можно предположить такой сценарий.
    1. Нет никакого горизонта событий
    2. Любое тело движется к какой-то точке, становясь все плотнее.
    3. Для нас это выглядит как застывший космонавт, как на субсветовой скорости.
    4. Для космонавта — некоторое весьма короткое время падения, а затем большой взрыв.
    5. Ни нас ни космонавта — нет и есть новая вселенная.


  1. steam13
    15.08.2016 11:52
    -1

    Интересно, а какого бы цвета была черная дыра, если бы свет все же мог вырваться?


    1. Capineiro
      15.08.2016 14:59

      Видимо цветом вырвавшегося света)


  1. Ainvain
    15.08.2016 11:52

    Физика черных дыр мне, как не умудренному глубинным пониманием физики обывателю очень непонятна. Позвольте задать пару вопросов:
    1. Насколько я понял — горизонт событий — это область, из которой объект не может покинуть гравитационную воронку ЧД, даже двигаясь со скоростью света. Т.е свет от него не достигает внешнего наблюдателя. Ок. Однако мне неясно, почему это означает, что из этой области невозможно ни передать информацию, ни выбраться. По поводу передачи информации — свет, который испускает тело, по моему пониманию может ВЫЙТИ за горизонт событий и быть зафиксированным оборудованием вне этого горизонта, но находящемуся в гравитационной воронке ЧД, на орбите. Ведь энергию свет полностью потеряет до выхода из грав. воронки ЧД, но после выхода из горизонта событий. Где-то между. Там мы его засекаем и передаем дальше. В данном случае я мыслю сугубо корпускулярно, на остальное мой мозг не покушается. Как пример — если мы бросим объект вертикально на Земле, он не покинет её гравитационную воронку. Представим, что некая область вокруг Земли — горизонт событий, а вторая космическая — наша аналогия скорости света. Мы можем запустить тело со скоростью, позволяющей выйти за пределы этой области, но тело всё равно упадет. Там мы тело сталкивается с детектором — детектор передает информацию за пределы грав. воронки с помощью такого же тела. Миссия выполнена. Энергия детектора уменьшена за счет передачи (или же просто ускорения имеющегося) носителя информации. Но информация покинула горизонт событий.
    2. Перефразируем проще — объект может покинуть горизонт событий, двигаясь с ускорением, превышающем ускорение свободного падения на горизонте событий. Соответственно, рукотворный объект может покинуть горизонт событий, используя реактивную тягу? И унести информацию.
    3. Соответственно и невозможность использования троса непонятно. С чего это материалу троса разрушаться? Автор представил в виде доказательства неизбежность передачи глюона от одного кварка к другому. Почему это невозможно. Если один кварк ближе к сингулярности, другой дальше — часть энергии глюона будет потеряна на преодоление гравитации, но он всё равно долетит, ведь его задача — не покинуть ЧД, а перенести взаимодействие внутри её.

    Сорь за многабукаф.


    1. Capineiro
      15.08.2016 14:45

      Дело в том, что при приближении к ЧД время замедляется, Наблюдатель с земли никогда не дождется момента когда зонд с корабля долетит до ЧД. Корабль тоже в свою очередь не дождется этого момента, пока он летает на безопасной орбите, допускающей возвращение.


    1. Mad__Max
      16.08.2016 03:13

      Почему это невозможно для реальных ЧД показать очень просто. Действительно нужно «всего лишь» двигаться с ускорением выше ускорения свободного падения. Все соль в том что для ЧД это ускорение стремится к бесконечности. Хотя конкретно на горизонте оно еще не бесконечное, но слишком большое с любой практической точки зрения. Например для ЧД массой в 10 раз больше солнечной это будет 1516805515656 м/с^2 — в 150 миллиардов раз больше чем на Земле.
      В случае троса (а не реактивной тяги которая тоже нереальна) возникающие в нем усилия в нем просто будут на много порядков выше прочности межатомных связей любого даже идеального материала. В результате трос не просто порвется — его как минимум расщепит на отдельные атомы каждый из которых будет продолжать падать уже независимо от других.

      Но вот чисто теоретически если пофантазировать — можно представить настолько гигантскую ЧД, что эти ускорения и силы на горизонте событий будут оставаться на относительно вменяемом уровне. Таких правда скорее всего вообще нет в нашей вселенной — например крупнейшая ЧД у нас галактике с массой больше 4 миллионов солнечных должна иметь ускорение свободного падения на своем горизонте порядка 380 000 земных, но вот чисто теоретически не понятно что помешает проделать мысленный трюк с тросом или передачей сигналов «по цепочке» на горизонте неограниченно большой ЧД.


      1. Capineiro
        16.08.2016 08:36

        Поясните пожалуйста, почему по нормали из гигантской ЧД даже фотон не улетит, а по спирали хоть и с чудовищным ускорением 380 000 g это может получиться?


        1. viatro
          16.08.2016 23:23

          Из-за эффекта "увлечения инерциальных систем отсчёта". Только не из самой черной дыры, а из области пространства вокруг вращающейся черной дыры, называемого эргосферой. В ней любой объект вынужден вращаться вокруг черной дыры в ту же сторону, что и сама черная дыра, а оставаться на месте можно только двигаясь со скоростью света в противоположную вращению сторону. Таким образом, двигаясь с достаточным ускорением по нормали от такой черной дыры объект на самом деле будет двигаться по раскручивающейся спирали. Для невращающейся черной дыры такого эффекта нет.


          1. Capineiro
            17.08.2016 09:22

            Эргосфера — это все-таки над горизонтом. Вопрос относился к «но вот чисто теоретически не понятно что помешает проделать мысленный трюк с тросом или передачей сигналов «по цепочке» на горизонте неограниченно большой ЧД.»


        1. Mad__Max
          18.08.2016 01:29
          +1

          Потому что фотон летит «по инерции» (без ускорения), тогда как наше гипотетическое тело (убер звездолет/зонд) летит хоть и с несколько меньшей скоростью, но зато с ускорением. Если это ускорение будет больше создаваемого сингулярностью — он по-идее должен вылететь из под горизонта. Ну если не помешают еще какие-то неизвестные сейчас физические явления, появляющиеся в этих экстремальных условиях и о которых мы еще не знаем.

          В ОТО гравитация не является силой и не создает каких либо ускорений. И притяжение тел друг к другу или скажем падения одного объекта на другой (ЧД к примеру) — это на самом деле тоже простое движение по инерции, просто в искривленном пространстве-времени. Объекты просто летят по геодезическим линиям, которые заменяют собой прямые линии траекторий которые были бы при полете по инерции в не искривленном пространстве.

          Невозможность вылететь из ЧД формулируется как то, что все геодезические линии в ней ведут к центру независимо от скорости объекта.

          Но как только на объект воздействует какая-то сила, у него появляется ускорение — он больше уже не двигается по геодезическим линиям, т.к. летит уже НЕ по инерции. (в обычном не искривленном пространстве — ускоряющийся объект совершенно не обязан лететь по прямой, он может лететь по какой угодно траектории — на какую хватит создаваемого ускорения). И так до тех пор пока ускорение не перестанет действовать.

          Ну или в другой формулировке. Считается что ЧД невозможно покинуть, т.к. для нее 2я космическая скорость превышает скорость света. А ничто не может превысить скорость света. Значит вроде как ничто не может покинуть ЧД.

          Но чтобы покинуть гравитирующий объект превышение 2й космической скорости вообще-то не является обязательным условием! Просто это самый простой способ выйти из области его притяжения. 2я космическая это эквивалент начальной скорости которую нужно сообщить 1му телу, чтобы он вышел из притяжения 2го тела при условии что дальше (после получения этой начальной скорости) 1е тело будет двигаться чисто по инерции(без собственного ускорения и каких либо внешний действующих на него сил).
          Однако если ускорение у 1го тела присутствует не только в начальный момент времени, но и дальше в процессе всего полета — то можно покинуть 2е тело не разгоняясь выше его 2й космической скорости.

          Давайте для пример посмотрим на Землю. 2я космическая для поверхности Земли примерно 11.2 км/с. Самый простой способ полностью покинуть притяжение Земли — разогнаться выше 11.2 км/с после чего двигатели можно вообще выключать и лететь дальше по инерции.

          Но есть и другой путь. Например:
          1. Строим эпический космический лифт и начинаем подниматься по нему, хоть со скоростью всего 1 м/с. Поднимаемся, удаляемся насколько угодно в космос, а скорость относительно центра Земли так 1 м/с и не превышает. Ограничения тут технические (как построить такую конструкцию, из чего ее делать — прочности материалов скорее всего никаких не хватит и т.д.), но не теоретические.
          2. Строим космический корабль с очень эффективными двигателями — которые могут создавать тягу достаточную, чтобы придавать кораблю ускорение больше 1g в течении очень длительного времени. И взлетаем не по касательной к поверхности пытаясь как можно быстрее набрать максимальную скорость — как летают современны ракеты, а перпендикулярно поверхности и не развивая больших скоростей. Т.е. даем двигателям такую тягу, чтобы ускорение было лишь чуть-чуть выше 1g, но зато выдерживаем его в течении очень долгого времени. Это будет очень неэкономный вариант полета(а на текущих технологиях и просто невозможный) — но так можно улететь далеко от поверхности планеты не превышая 2ю космическую скорость.

          Ну а теперь обратно к нашим черным дырам и космических кораблям сверхразвитых цивилизаций. Если такой корабль каким-то образом сможет длительное время развивать двигателями ускорение выше ускорения свободного падения ЧД (например выше 380 000 g для центральной ЧД нашей галактики) и при этом оставаться целым, то он сможет удаляться от ЧД не превышая ее 2ю космическую скорость, т.е. не выходя за ограничения накладываемые скоростью света.
          Практически это конечно нереально — невозможно создать движки создающие такую мощную тягу и не существует даже теоретически материалов из чего создать сам корабль, чтобы он выдерживал такие ускорения.

          Но чисто теоретически — если рассматривать все более и более массивные ЧД, то в какой-то момент эти ускорения и силы станут вполне вменяемыми — например сравнимыми с ускорением свободного падения на поверхностях планет. Это будет уровень где-то порядка 100 миллиардов солнечных масс и выше. Т.е. ЧД массой в целую крупную галактику. Таких скорее всего тоже нет на практике, но опять же теоретических ограничений на размер ЧД нет, так что в теории можно рассматривать и настолько большие ЧД.


          1. Capineiro
            18.08.2016 09:24

            Все равно остается непонятным следующий момент. Пусть космический корабль по нормали улетает немного из-под горизонта с ускорением 380 001 g. Прожектор на носу корабля в момент начала движения послал вперед фотоны, которые начали удаляться относительно корабля со скоростью света. Если таким образом корабль их когда-нибудь догонит, Значит в этот момент единственным вариантом который я вижу хоть как-то осмысленным, фотон будет лететь навстречу (обратно) к кораблю со скоростью света. Такая инверсия кажется очень странной с учетом того что фотон собирался относительно корабля улететь на бесконечность и в каждый момент времени вроде бы к этому и стремился.


            1. Mad__Max
              20.08.2016 00:56

              Корабль никогда свет не догонит — свет хоть чуть-чуть но всегда будет быстрее двигаться. Точнее в системе отсчета удаленного наблюдателя — чуть-чуть быстрее. А в системе отсчета (с точки зрения) корабля — он будет как положено со скоростью света от корабля улетать.

              Разница в том, что в отличии от корабля свет вылетая из ЧД будет испытывать все большее и большее красное гравитационное смещение. Теряя из-за этого энергию (длины волн растут/частота снижается). И уже на самом горизонте ЧД(если источник находился точно на горизонте в момент излучения) это смещение достигает бесконечности — т.е. свет потеряет всю энергию и до удаленного наблюдателя просто не дойдет — от него ничего не останется (длина волны стремится к бесконечности — колебания ЭМ-поля прекратились, т.е. волны уже в общем-то не существует). Т.е. с точки зрения удаленного наблюдателя — свет из ЧД так и не выбрался и остался в ней навсегда.

              Из-за этого немного выше кто-то приводил еще другой мысленный эксперимент — с цепочкой из кораблей/ретрансляторов, передающих друг другу по лучу данные и пересылающие их дальше по цепочке(при этом у них может быть даже недостаточно тяги чтобы компенсировать притяжение ЧД — достаточно чтобы они падали в ЧД не слишком быстро).
              При каждой такой ретрансляции исходная частота/длина волны будет восстанавливаться. И поэтому хоть исходный луч света(или радиоволна к примеру) из ЧД выбраться и не сможет — но вот информация которую он нес сможет из ЧД «сбежать» и быть принятой удаленным наблюдателем. Т.е. теоретически свехмассивную ЧД с «чистым» окружением можно «прощупать» вплоть до горизонта событий включительно, если скинуть в нее сразу цепочку из зондов — первый из которых будет обвешен датчиками и собирать данные, а остальные ретранслировать его сигнал компенсируя гравитационная смещение, чтобы наблюдатель на корабле смог его принять.

              А вот хотя бы чуть-чуть под горизонт уже непонятно что будет происходить — красное смещение должно стать еще больше. Но куда еще больше чем в бесконечность раз? Разве что фотоны должны приобрести отрицательную энергию…
              В общем очередное наглядное указание на то, что ОТО под горизонтом ЧД уже не работает и нужна другая теория, которая ее заменит увеличив границы применимости, как ОТО заменила(расширила) Ньютоновкую физику. ОТО точно не окончательная и не полностью верная теория. Просто пока никто не смог предложить что-то получше.


              1. napa3um
                20.08.2016 11:10

                «Cвет хоть чуть-чуть но всегда будет быстрее двигаться» — не чуть-чуть, а будет двигаться со скоростью света относительно любой системы координат.


                1. Mad__Max
                  20.08.2016 15:56

                  Относительно любой системы координат, но не относительно любого объекта в этой системе координат.
                  Речь выше шла о том, что если корабль летит со скоростью близкой к скорости света и выпускает луч света, то с точки зрения далекого неподвижного наблюдателя — они (и корабль и луч света) будут лететь с близкими скоростями — свет лишь немного быстрее. (например корабль со скоростью 0.99с, а луч с 1с)

                  А вот с точки зрения корабля (в системе координат в которой корабль неподвижен) — луч будет двигаться от него со скоростью света.


              1. Capineiro
                20.08.2016 14:39

                Допустим корабль летит над горизонтом и в принципе наблюдаем снаружи. Он выпустил зонд, который улетел немного под горизонт и отправил на корабль сигнал. В результате этого сигнал ослабел в N раз (длина волны увеличилась в N раз). Корабль усилил этот сигнал в N раз (восстановил длину волны) и ретранслировал его на землю. Profit. Теперь поставим на второй зонд лампочку с длинной волны в 100500 N раз короче. Даже лучше: в N^100500 раз короче Разумеется до корабля долетит сигнал не слабее чем тот, который он ретранслировал на землю в прошлый раз. Поэтому можно обойтись без корабля. Фотоны из под горизонта напрямую добрались до Земли. Противоречие с определением горизонта. Поэтому предположение о вылетающем из ЧД растянувшимся до бесконечности фотоне неверно.
                Логичнее звучит, что корабль не будет получать ни от первого ни от второго зонда никакого сигнала, как только они уйдут под горизонт


                1. Mad__Max
                  20.08.2016 16:34

                  Противоречия с горизонтом нет. Написал — до горизонта включительно, но не дальше его. От источника за горизонтом ослабление теоретически должно составить бесконечное кол-во раз (N стремится к бесконечности уже в момент пересечения горизонта), поэтому насколько сигнал не усиливай — он все-равно не дойдет.

                  Впрочем эта бесконечное красное смещение из разряда прочего множественного бреда генерируемого несовершенством ОТО если ее пытаться применять там, где она уже не работает. Нужна новая нормальная теория, типа теории квантовой гравитации, чтобы сказать что в этом случае будет происходить со светом на самом деле.


                  1. Capineiro
                    20.08.2016 17:27

                    До горизонта включительно да, тогда согласен.
                    А в чем бред бесконечного красного смещения? Для меня логично что если ЭМ поле колеблется бесконечно медленно (т. е. не колеблется), то и длина волны при этом становится бесконечной большой (равносильно её отсутствию). Думаю ОТО перестает работать не на горизонтах галактических ЧД, а экстремально близко к их центрам-сингулярностям, где кривизна пространства и приливные силы становятся умопомрачительными.


                    1. Mad__Max
                      20.08.2016 22:11

                      В том что в реальной природе в отличии от математики не бывает ничего по настоящему бесконечного. Кроме может быть самой вселенной — и то это под большим вопросом.

                      Само то бесконечное красное смещение это лишь следствие — основной затык это бесконечное замедление времени из-за которого и будет наблюдаться бесконечное красное смещение. И это бесконечное замедление и генерирует бесконечное количество бреда.
                      Бесконечное красное смещение для внешнего наблюдателя. И наоборот бесконечное синее смещение для падающего в ЧД.

                      С точки зрения внешнего наблюдателя согласно формулам ОТО — в ЧД упасть в принципе невозможно — все объекты застрянут у самого горизонта в момент его касания (точнее даже и коснуться-то по настоящему не смогут — только стремиться к нему в пределе).
                      А с точки зрения падающего в ЧД согласно той же ОТО — вообще никакого горизонта не будет и он пролетит его даже не заметив — а все неприятности начнутся только при приближении к сингулярности из-за стремящихся к бесконечности приливных сил. Которой сингулярности при этом на самом деле не существует — до центра ничего еще упасть не успел с момента рождения вселенной, так что сингулярность еще просто не сформировалась.
                      А значит что пока он падает в ЧД(причем по своим меркам это будет очень быстро — доли секунд для звездых ЧД, минуты-часы для сверхмассивных ЧД) — вся остальная вселенная умрет от старости.

                      А раз вселенная умрет от старости — значит он никогда до сингулярности так и не долетит: т.к. во внешней вселенной пройдет бесконечное время, то пока объект падает она успеет остыть до почти абсолютного нуля, а ЧД в которую он падает успеет испариться за счет излучения Хокинга. Это будет происходить очень долго — минимум триллионы лет. Но какая разница? У нас же бесконечное замедление времени — так что обязательно дождемся!

                      Скорость опять же — если горизонт он все-таки как-то пересечет, то скорость свободно падающего объекта превысит скорость света — что невозможно. Правда на это приводится обычно контраргумет — что дескать т.к. внешние наблюдатели этого не увидят, и вернуться и рассказать он никогда не сможет — то парадокса вроде как бы и нет. Выглядит как детская отмазка — пока никто не видит, можно законы вселенной нарушать получается.
                      И т.д.

                      Уравнения эйншейна можно пытаться решать и за горизонтом событий при помощи разных ухищрений. И это неоднократно делалось. Только в зависимости от трактовок и допущений получаются совсем разные варианты у разных авторов (ученых).
                      Где-то и путешествия во времени вылезают. Где-то мосты в другие пространства (измерения/вселенные) — кротовые дыры и мост Эйнштейна — Розена. И много чего еще другого, что скорее никакого отношения к реальности не имеет.


                1. black_semargl
                  20.08.2016 20:43

                  А не поставить такую лампочку — потому что излучённый фотон чтобы улететь должен содержать энергии больше чем масса этой лампочки.
                  А ещё чуть ближе к горизонту — больше чем масса корабля.
                  А совсем близко — больше чем масса ЧД
                  Но гораздо раньше у нас сам фотон в ЧД превратится.


          1. Capineiro
            18.08.2016 13:04

            Чтобы начать удаляться с ускорением 380 001g от горизонта галактической ЧД, корабль должен иметь начальную моментальную скорость относительно локального пространства равной скорости света. И только ускорение 380 000g позволило бы ему удерживать эту скорость постоянной. Но очевидно, что это невозможно


            1. Mad__Max
              20.08.2016 02:25

              Не нужно ему иметь скорость равную скорости света. Это если он пытается удержаться на орбите вокруг нее, не компенсируя двигателями хотя бы часть притяжения от сингулярности — тогда да нужна. Хотя даже в этом случае не нужно именно скорость света развивать, а просто очень высокую скорость — чтобы перестать падать на гравитирующий объект (чтобы расстояние до его центра не увеличивалось и не уменьшалось) — достаточно 1й космической скорости.
              Для пространства у горизонта ЧД 1я космическая составляет примерно 0,7 скорости света(корень из 1с). Т.е. чтобы иметь возможность удаляться от горизонта событий в принципе достаточно развить скорость > 0.7c.

              1c понадобится только если мы хотим не просто удаляться от этой поверхности, но улететь от нее произвольно далеко («на бесконечность» — полностью выйти из грав. поля этого объекта и больше к нему уже не возвращаться) и при этом не используя дополнительное (не связанное с гравитацией) ускорение.

              И это все для случаев пролета по касательной. Если у нас наш виртуальный корабль настолько крут, что может поддерживать нужную тягу все время сближения с ЧД, то он может не на большой скорости проскакивать у горизонта, а опускаться к горизонту вообще по нормали — постепенно, по мере приближения постоянно увеличивая тягу, чтобы всегда почти полностью компенсировать притяжение. Тогда скорость при подходе к горизонту у него будет околонулевая и он может буквально «зависнуть» над горизонтом не двигаясь относительного него или центра ЧД (сингулярности). А чтобы начать удаляться от него — нужно лишь еще чуть прибавить тяги, чтобы ускорение превысило свободное падение. И удаляться так же можно с произвольной скоростью.

              Все отличие от пролета по касательной на скорости >0.7 c будет только в на порядки большем расходе энергии и «рабочего тела» (ли за счет чего у нас там двигатели корабля будут создавать тягу). Проскочить на скорости намного «легче» — тем более большую часть этой скорости сама ЧД и даст по мере приближения к ней.


              1. Capineiro
                20.08.2016 14:13

                Фотонная сфера находится на расстоянии в полтора раза выше горизонта. На фотонной сфере кораблю уже необходимо лететь на скорости не менее скорости света, чтобы удерживать орбиту и не начать падать. Достигнуть этого уже невозможно даже с суперкораблем. Это несколько расходится с достаточной 0.7с на горизонте.

                Всё же траектория корабля зависит не только от ускорения, но и от начальной скорости. Если спутник сошел с орбиты земли и начал падать вниз, чтобы улететь обратно, ускорения 2g ему может не хватить (упадет на землю раньше, чем моментальная скорость сравняется с нулём). Чтобы начать улететать с ускорением 380 001g от горизонта надо сначала умудриться компенсировать свою световую скорость свободного падения на ЧД, на что никакой горючки не хватит.


                1. Mad__Max
                  20.08.2016 18:55

                  Ну да, тут похоже моя ошибка. 2я космическая на горизонте событий = скорости света это по классической физике получается для объекта такого размера(радиус = гравитационному радиусу) и массы. По ОТО выходит что у горизонта даже 1я космическая немного больше скорости света.
                  Правда не очень понимаю какая конкретно из поправок ОТО приводит к тому, что 1я космическая для ЧД в корень (3) раз выше чем для классических объектов той же самой массы. Фотосфера (находящаяся в 1.5 гравитационных радиусах от сингулярности) это как область где 1я космическая =1с. 2я космическая там впрочем тоже = 1с. «зазора» между 1й и 2й как у всех «нормальных» космических объектов тут нет.

                  Точнее понятие 1/2 космической скорости к ЧД вообще малоприменимы — они теряют свой изначальный смысл. Посмотрел еще раз формулы ОТО. С этой мозговыворачивающей теорией(если ее пытаться применять к экстремальным случаям) получается совсем забавно:
                  — если лететь по касательной к горизонту событий, то уже в 1.5 радиусах аналог 1й космической будет равен скорости света, кстати 2я космическая тоже будет равна скорости света. т.е. 1я и 2я равны
                  — если лететь по нормали к горизонту, то аналог 2й космической доходит до 1с только непосредственно на самом горизонте и меньше 1с если его не касаться, а лишь сколь угодно близко к нему подойти

                  Тогда как для обычных объектов (чей грав. радиус много меньше реального радиуса) разницы нет — скорости в обоих случаях равны и не зависят от траектории, а вот зато 1я и 2я космические различаются в корень(2) раз.

                  И забавные следствия:
                  — луч света (или любой материальный объект) летящий по касательной к ЧД и прошедший ближе 1.5 грав. радиусов от ее центра уже никогда от ЧД не уйдет и неизбежно падает в нее.
                  — луч света выпущенный по нормали из области около(но хотя бы чуть-чуть не доходя) самого горизонта (1.001 грав. радиус к примеру) или даже материальное тело подкинутое вверх со скоростью очень близкой к 1с — из гравитации ЧД уйдет и свободно улетит (случаи столкновения с чем-то падающим навстречу не рассматриваем — если окрестности ЧД чистые)
                  — объект обладающий скоростью ниже 1с, но имеющий собственное ускорение может по касательной проходить у ЧД и возвращаться из областей из которых свет вернуться уже не может. ускорение при этом естественно должно быть постоянно направлено не на дальнейший разгон по направлению движения (это бесполезно), а по нормали к горизонту, как бы не давая гравитации заворачивать вектор скорости к центру ЧД

                  Про траекторию понятно — но я писал в прошлом сообщении, что если мы хотим пролететь у горизонта на маленькой скорости или вообще «зависнуть» над ним, то тормозить двигателями нужно конечно не только в самый последний момент(в этом случае потребуется ускорение стремящееся к бесконечности, что невозможно) — а все время пока идет сближение. Начиная с малой тяги и маленьких ускорений (пока находимся далеко от горизонта), но постепенно ее наращивая. Так чтобы прирост скорости от гравитации ЧД компенсировался снижением скорости от создаваемой тяги, а скорость сближения была примерно постоянной. Ну или сначала полпути свободного падение (с увеличением скорости) и полпути торможения. Или другие промежуточные варианты — смотря какое максимальное ускорение может корабль развивать.

                  Хватит топлива или нет это уже технические детали, мы в любом случае выдуманный корабль обсуждаем, а не какой-то реальный. Ну и 380 000g — это для не очень большой конкретной ЧД в центре нашей галактики. Тоже писал что если брать действительно гигантские ЧД, то там ускорение свободного падения может быть к примеру и 1-2g всего и тогда каких-то нереальных характеристик от подобного корабля не потребуется. Наших химических ракетных движков конечно не хватит, но какой-нибудь сверхэффективный и мощный плазменный двигатель запитанный хотя бы от атомного или термоядерного реактора — справится. Не говоря уже о технологиях, которыми может обладать какая-нибудь сверхразвитая цивилизация и о которых мы пока не знаем даже в теории.


                  1. Capineiro
                    20.08.2016 20:14

                    Правда не очень понимаю какая конкретно из поправок ОТО приводит к тому, что 1я космическая для ЧД в корень (3) раз выше чем для классических объектов той же самой массы.

                    Все-таки в из-за сжатий и замедлений у горизонта, значения космических скоростей у экипажа и астронома землянина могут отличаться (разные системы отсчета)
                    — если лететь по касательной к горизонту событий, то уже в 1.5 радиусах аналог 1й космической будет равен скорости света, кстати 2я космическая тоже будет равна скорости света. т.е. 1я и 2я равны
                    — если лететь по нормали к горизонту, то аналог 2й космической доходит до 1с только непосредственно на самом горизонте и меньше 1с если его не касаться, а лишь сколь угодно близко к нему подойти
                    — объект обладающий скоростью ниже 1с, но имеющий собственное ускорение может по касательной проходить у ЧД и возвращаться из областей из которых свет вернуться уже не может. ускорение при этом естественно должно быть постоянно направлено не на дальнейший разгон по направлению движения (это бесполезно), а по нормали к горизонту, как бы не давая гравитации заворачивать вектор скорости к центру ЧД

                    Похоже на правду

                    Начиная с малой тяги и маленьких ускорений (пока находимся далеко от горизонта), но постепенно ее наращивая. Так чтобы прирост скорости от гравитации ЧД компенсировался снижением скорости от создаваемой тяги, а скорость сближения была примерно постоянной.

                    Это эквивалентно тому, что начиная с малой тяги и малой скорости, достичь околосветовой скорости.

                    Тоже писал что если брать действительно гигантские ЧД, то там ускорение свободного падения может быть к примеру и 1-2g всего и тогда каких-то нереальных характеристик от подобного корабля не потребуется. Наших химических ракетных движков конечно не хватит, но какой-нибудь сверхэффективный и мощный плазменный двигатель запитанный хотя бы от атомного или термоядерного реактора — справится.

                    Да, если горизонт не пересекли еще, то есть шанс улететь на малых g. Только горючки все равно потребуется столько, что она сама в черную дыру схлопнется от своего же веса)))


                    1. Mad__Max
                      20.08.2016 23:45

                      Все-таки в из-за сжатий и замедлений у горизонта, значения космических скоростей у экипажа и астронома землянина могут отличаться (разные системы отсчета)

                      Да, я правда все для удаленного смотрел. Но похоже на то что это просто очередной эффект замедления времени. С 1й/2й космической все в порядке. — это просто сама скорость света(именно света как конкретной электромагнитной волны, а не фундаментальная константа взаимодействий) в сверхсильном гравитационном поле ниже.
                      Не знаю имеет ли это именно такой физический смысл в реальности, но численно фотосфера ЧД находится в 1.5 гравитационных радиусах от центра ЧД потому что на этом расстоянии 1я космическая и скорость света с учетом замедления времени сравниваются.
                      1я космическая тут как и для любых обычных тел будет около 0.577 скорости света, 2я космическая ~0.82 c
                      А у горизонта 1я ~0.71с, 2я = 1с

                      Для света 1.5 радиуса являются особым случаем, т.к. на таком расстоянии (1.5 от центра, 0.5 от горизонта событий) скорость самого света из-за действия гравитации как раз равняется 0.577с. В 1.732 раза ниже скорости света в вакууме и при отсутствии сильных гравитационных полей. Так же и красное смещение для этой поверхности будет Z = 0.732, т.е. длины всех волн испущенных на расстоянии 1.5 радиусов будут для удаленного наблюдателя увеличиваться в 1.732 раза.

                      На практике это наблюдается как одно из следствий ОТО: Эффект Шапиро — свет проходя мимо тяжелых тел не просто искривляет свою траекторию, но и проходит мимо них медленнее чем он шел бы в пустом пространстве.

                      Поэтому чтобы «поймать и не отпускать» пролетающий мимо свет даже ЧД необязательна — очень плотный объект, но еще не достигший критической плотности будет обладать теми же свойствами.

                      Это эквивалентно тому, что начиная с малой тяги и малой скорости, достичь околосветовой скорости.

                      Не эквивалентно. Точнее было бы эквивалентно с точки зрения ньютоновской физики, но не эквивалентно согласно ОТО. Т.к. реально у нас скорость все время маленькая(много меньше чем скорость света) — то нет эффектов замедления времени (тут имеются ввиду эффекты СТО — связанные со скоростью движения объекта, а не от гравитационного поля/искривления пространства в котором он находится) и нет стремления массы+энергии (релятивисткой массы по старой терминологии) тела к бесконечности. А именно это не дает разогнаться до 1с в СТО в отличии от ньютоновской физики, т.к. потребуется бесконечная энергия и бесконечное время на разгон.

                      Тут же расход энергии/рабочего тела при спуске к горизонту на малых скоростях будет примерно аналогичен разгону до 1с по ньютоновской физике или ~0.61с по СТО. Т.е. конечно до… и скорее всего нереально на практике — но совсем не бесконечно много и поэтому теоретически не запрещено.


  1. golkond
    15.08.2016 11:54

    Хм… поправьте, где не прав, пожалуйста: если основной тезис — кварки не могут обмениваться глюонами, потому что те летят по геодезическим линиям и «не попадают» другой кварк из-за отклонения, вызванного кривизной пространства, — то отчего они «попадают» при меньших «отклонениях»? ведь пространство искривлено в любой точке любой другой присутствующей массой.
    И как формируется разница «попал-не попал»?


  1. EngineerArt
    15.08.2016 11:58

    Исходя из того что объект, преодолевая горизонт событий большой ЧД, не раскладывается на атомы, но свет обратно не возвращается, быть может стоит датчик повесить на кабеле?
    И передавать электрический сигнал.


    1. Capineiro
      15.08.2016 14:52

      Это верно только для свободнопадающего объекта. Какой смысл в свободно падающем кабеле? Электрический сигнал по кабелю кстати идет несколько медленнее чем фотон в вакууме.


      1. EngineerArt
        15.08.2016 15:10

        скорость раскручивания кабеля не такая высокая, можно что-то успеть передать


        1. Capineiro
          15.08.2016 15:13

          Кабель будет раскручиваться бесконечно долго до пересечения горизонта с точки зрения корабля, несущего катушку. Порвется он быстрее. Ну либо кораблю надоест и он улетит.


          1. EngineerArt
            15.08.2016 15:15

            С чего бы ему рваться?


            1. Capineiro
              15.08.2016 15:24

              Из-за приливных сил. От собственного веса. Но даже если он бесконечно прочный, падать на горизонт он всё равно будет бесконечно долго. Зачем тогда весь эксперимент?


  1. simki
    15.08.2016 14:46

    Еще интересное наблюдение. Насчет того чтобы попасть внутрь ЧД всё сложно. А вот как насчет того, чтобы изначально жить внутри ЧД? Давайте представим себе целую галактику (или лучше скопление галактик). Но не как нашу, а поплотнее, больше газа между звездами. Больше плотность галактики, выше искажения пространства-времени. В какой-то момент галактика может набрать вес (объединившись например, как наша с туманностью Андромеды, будь они плотнее) и превратится в черную дыру. Но наблюдатели внутри ничего не заметят. Они просто станут жить под неким куполом, ненаблюдаемым изнутри. А гравитация внутри ни как не изменится. Снаружи появится сфера Шарцшильда, но для местных жителей это будет математическая абстракция, ни как не наблюдаемая. Искажения их пространства буду сказываться на расстоянии под 100 000 световых лет, и будут слабее, чем искажения вызываемые нашей планетой. Локально будет всё то же самое. Возможно, они захотят поставить обратный эксперимент, перед захлопыванием ЧД запустить зонд наружу (момент определят, вероятно, по мощному «синему» смещению), а после пытаться получить с него данные. Зонд будет слать информацию в ЧД, но скорее всего внутрь она не попадет. Или попадет в странной форме, одиночный кванты от антенны превратятся в гамма кванты и далее во что-то странное, вот тут то самое интересное, что это будет.


    1. Capineiro
      15.08.2016 14:57

      В общем-то мы так и живем. По оценкам, радиус Шварцшильда для массы нашей вселенной превышает её наблюдаемую часть


      1. simki
        15.08.2016 15:14

        Ну тогда могут быть какие-то артефакты, прилетевшие снаружи. Например частицы высоких энергий, летящих на скорости 0.999(9) от скорости света, про них тут была статья, единичные частицы, обладающие энергией пули. Очень редкие. Возможно это случай предельного синего смещения.

        Может что-то связанное с темной материей, темной энергией. Частицы уже не «материальны», так как прошли сферу Шварцшильда, но обладают энергией, вот она так и проявляется, просто мысли в качестве гипотезы.


        1. Capineiro
          15.08.2016 15:27

          Всё что долетает, летело всего 13-14 млрд лет. Значит до этого времени ничего не летело